You are on page 1of 133

Paediatrics (No=385)

What is the most common cause of unilateral childhood exophthalmos?


orbital hemorrhage
orbital cellulitis
thyroid-related orbitopathy
capillary hemangioma

Your answer was CORRECT


Explanation
The most common cause of unilateral childhood exophthalmos is orbital cellulitis, secondary to
ethmoid sinusitis or respiratory tract infections.

All of the following clinical findings can be associated with Goldenhar's syndrome EXCEPT:

eyelid colobomas

lipodermoids

proptosis
Duane's syndrome

Your answer was CORRECT


Explanation
Figure: Goldenhar's syndrome

Goldenhar's syndrome is characterised by:

 preauricular appendages
 aural fistulas
 maxillary or mandibular hypoplasia
 hemifacial microsomia
 vertebral deformities (e.g. scoliosis)
 notching of the upper eyelid
 Duane's syndrome
 limbal dermoids
 sensorineural deafness
 other: microphthalmia, anophthalmia, ON hypoplasia, ocular colobomas

Note: proptosis is not a feauture of Goldenhar's syndrome. Proptosis is typically associated with
the synostosis syndromes, which include both Crouzon's and Apert's syndromes.

Which is a typical characteristic of JIA-associated uveitis?

mutton-fat keratic precipitates

retinal vasculitis

hypopyon

posterior synechiae

Your answer was CORRECT


Explanation
JIA-associated uveitis is typically non-granulomatous with no hypopyon. It can occur in a
relatively white, pain-free eye. Posterior synechae are common. Complications include glaucoma,
band keratopathy, cataract and CMO.

Retinoblastoma is caused by a defect on chromosome:

21

17

13

Your answer was CORRECT


Explanation
Retinoblastoma is caused by a defect in chromosome 13q14, the location of RB1 which is a
tumour suppressor gene.

An 8-year-old boy presents with 6/18 vision and exhibits radial, spoke-like disturbances at both
maculae on slit-lamp biomicroscopy. One of his uncles was similarly affected.

All of the following are true about this condition EXCEPT:


cleavage of the retina at the nerve fiber layer
selective attenuated of b-wave on electroretinogram
X-linked inheritance
classic petalloid leakage on fluorescein angiography

Your answer was CORRECT


Explanation
Juvenile X-linked retinoschisis is characterised by:
 cleavage of the retina at the nerve fiber layer
 inverted ERG (selective attenuation b wave)
 EOG and dark adaptation are normal or subnormal
 macular microcysts and radiating retinal folds, but no FFA leakage
 peripheral schisis usually develops later
 vitreous veils and strands can lead to vitreous haemorrhage

A 7-year-old boy presents with a sudden onset, rapid evolving unilateral ptosis over 1 week.
Examination shows significant oedema in the periorbital area with a palpable mass in the superior
nasal quadrant of the eyelid. Biopsy is performed immediately and the diagnosis of
rhabdomyosarcoma is made.

What is the best treatment?

chemotherapy alone

exenteration

radiation with or without additional chemotherapy

total surgical excision

Your answer was CORRECT


Explanation
Historically, exenteration was the primary treatment for rhabdomyosarcoma but this debilitating
procedure had a poor survival rate and has since been abandoned. Radiation and systemic
chemotherapy are now the mainstays of treatment. The total dose of local radiation varies from
4500 to 6000 rad given over a period of 6 weeks. Chemotherapy is used to eliminate microscopic
cellular metastasis. The survival rate using these modalities has improved significantly.

Early signs in the development of malignant hyperthermia include all of the following EXCEPT:

trismus

arrhythmia

elevated body temperature


darkening of the blood in the operative field

tachycardia

Your answer was CORRECT


Explanation
Elevated body temperature is a relatively late sign. Careful surveillance for the earlier signs is
critical.

Which statement regarding rhabdomyosarcoma is FALSE?


a common presentation is an orbital cellulitis-like picture

most common primary malignant tumor of the orbit in children

the average age at diagnosis is 7 years

when detected early, it may be appropriate to observe the lesion initially

Your answer was CORRECT


Explanation

Observation is not an option in rhabdomyosarcoma, shown above. Untreated, the condition is fatal
from extension into brain, meninges, lymphatics and blood.
The most common clinical finding in infants with congenital rubella syndrome is:
cataract
patent ductus arteriosus
sensorineural hearing loss
pigmentary retinopathy
mental retardation

Your answer was CORRECT


Explanation
Congenital rubella syndrome is characterised by:
 Classic Triad
o Sensorineural deafness
o Ocular abnormalities: cataract, glaucoma, pigmentary retinopathy
o Congenital heart disease: patent ductus arteriosus
 Other findings
o intrauterine growth restriction
o CNS abnormalities: mental retardation, microcephaly, meningitis
o hepatosplenomegaly
o skin, bone, haemtological abnormalities

Which one of the following is the LEAST important risk factor for developing ROP?

twins
birth weight less than 1250g

hyperoxia
gestational age

Your answer was CORRECT


Explanation
The major risk factors for developing ROP include hyperoxia, low gestational age, and low birth
weight.

Which of the following is a preferential looking visual test for use in children?

Teller

Sonksen

Sheridan-Gardiner

Kay
Your answer was CORRECT
Explanation
The Teller and Keeler cards are preferential looking cards containing gratings. Cardiff cards are
preferential looking cards, which contain simplistic images in light-grey (e.g. a duck) at one end of
a grey card. Preferential looking cards are useful for 0 months to 2 years.

The Kay, Sheridan-Gardiner, Songksen and Keeler logMAR tests contain pictures and/or letter
optotypes for matching or naming in single or crowded optotypes (good for 2+ years for Kay and
2-3+ years for other options).

In congenital nasolacrimal system obstruction, where is the most common site of obstruction?

lacrimal sac

lower canaliculus

punctum
common canaliculus

Valve of Hasner

Your answer was CORRECT


Explanation
Congenital nasolacrimal system obstruction is relatively common and is seen in 2% to 4% of
newborn infants. Blockage occurs at the valve of Hasner. In 80% to 90% of patients, the
membranous valve of Hasner opens spontaneously or with gentle massage of the nasolacrimal sac
within the first 6 to 12 months of life. If blockage persists after 12 months, probing is indicated.

A 13-year-old girl presents to eye casualty with sudden loss of vision 2 days ago in her right eye.
On examination, her right VA is hand movements, while left VA is 6/6. Her pupil reactions are
normal, with no RAPD. Slit lamp examination, including dilated fundoscopy is normal. OCT and
FFA imaging are normal in both eyes.

Given the history, what is the most likely finding when a 20-dioptre base-out prism is placed
before the right eye?

no movement

movement of both eyes towards the left followed by a corrective adduction movement of the
left eye

movement of both eyes towards the right initially

movement of both eyes towards the left without a corrective movement of the left eye
Your answer was CORRECT
Explanation
The history in this case is suggestive of functional visual loss. The 20-dioptre base-out prism test
is useful in helping to confirm functional visual impairment, where one would expect normal (as
opposed to pathological) reactions to the prism. The normal movement when a 20-dioptre base-out
prism is placed in front of an eye is for both eyes to move towards the apex of the prism, followed
by an adduction movement of the fellow eye to take up binocular fixation. In genuine, dense
monocular visual impairment, one would expect no movement of the eyes when the prism is
placed in front of the affected eye.

This question came in the 2014 FRCOphth.

The most common ophthalmic finding in the craniosynostosis syndromes is:

ankyloblepharon

hypertelorism and proptosis

large orbit with apparent enophthalmos


blepharophimosis and ptosis

Your answer was CORRECT


Explanation

Figure: Apert syndrome (left) and Crouzon syndrome (right)

The orbits of patients with craniosynostoses (e.g. Crouzon's, Apert's and Pfeiffer's syndromes) are
usually shallow and small, with resultant proptosis and exposure. V-pattern exotropia is also
common.
All of the following orbital lesions are associated with increased proptosis with the Valsalva
maneuver or crying EXCEPT?
orbital lymphangioma
orbital varix
capillary haemangioma
orbital cavernous haemangioma

Your answer was CORRECT


Explanation

Characteristics of orbital cavernous haemangioma:


 most common adult intra-conal lesion
 large blood-filled spaces
 not a true neoplasm (not from proliferating clonal cells)
 typically middle-age, women
 proptosis (usually axial)
 reduced vision, diplopia, lid swelling, palpable mass
 usually intra-conal (rarely involves orbital apex)
 does NOT enlarge with Valsalva (differentiating from varices, AVM, lymphangioma and
capillary haemangioma)
 pleboliths may be present (these also occur in other slow moving venous malformations such
as varices)
 imaging: lesions round, homogenous, encapsulated, not involving apex
 MRI: hypo-intense on T1, hyperintense on T2 (not unique)
 differential: rhabdomyosarcoma, meningioma, neurofibroma, neurilemmoma,
haemangiopericytoma (malignant)
 treatment: nil if asymptomatic, surgery (lateral orbitotomy) for excision

Which is FALSE regarding abnormalities of corneal size?


a defining criteria of megalocornea is a horizontal corneal diameter over 13 mm
eyes with isolated microcornea are generally hyperopic
megalocornea may be associated with Down's, Marfan's and Alport's syndrome
most cases of megalocornea are unilateral and seen in men

Your answer was CORRECT


Explanation
Most cases of megalocornea are bilateral and seen in men. Other statements above are true.

Megalocornea is characterised by:


 bilateral condition
 typically X-linked (male predominance)
 corneal diameter over 13 mm after 2 yrs of age
 corneal diameter over 12 mm in infants
 normal IOP (differentiates from buphthalmos)
 associated with Down's, Marfan's and Alport's

A 7-year-old boy presents with right leukocoria and right exotropia. Vision in this eye is reduced
to hand movements. On examination, there is anterior uveitis and vitritis. Fundal views are hazy
but there is a granuloma visible in the upper peripheral retina. There is a dense greyish-white
exudate all round the retina peripherally and vitreo-retinal traction bands.

Which infection is MOST likely:

presumed ocular histoplasmosis

onchocerciasis

cysticerosis

toxocariasis

Your answer was CORRECT


Explanation
Figure: ocular toxocariasis showing old foci of infection at the posterior pole with a tractional
band; during active disease significant vitritis is common

The presentation in the question above suggests chronic endophthalmitis due to toxocariasis. This
condition typically presents between 2 and 9 years of age with leukocoria, strabismus and visual
loss. There is anterior uveitis and vitritis. In some cases there is a peripheral granuloma. The
peripheral retina and pars plana are covered by greyish-white exudate.

Onchocerciasis is endemic to Africa and areas of South America. Ocular involvement is usually
bilateral and posterior segment findings include chorioretinitis with RPE atrophy and clumping.
Cysticercosis is caused by the pork tapeworm. It is more prevalent in China, India, Sub-Saharan
Africa and parts of South America. Ocular involvement involves cyst formation sub-
conjunctivally, free-floating in the AC, or sub-retinally. Presumed ocular histoplasmosis usually
presents in older patients (the second to fifth decades) and it occurs in the absence of intra-ocular
inflammation, with atrophic histo spots scattered in the mid-retinal periphery.

An 11-year-old boy has visual acuity of 6/6 in his right eye and 6/9+ in his left. Examination of
the left fundus reveals a yellow-orange circular, elevated, lesion centred at the fovea. The boy's
father reports that several members of his family including himself and a sister have suffered from
mild to moderate deterioration of vision since youth.

Which one of the following would be most appropriate in establishing the diagnosis?
visual evoked potentials
fluorescein angiogram
ocular ultrasound
EOG
chromosome analysis

Your answer was CORRECT


Explanation

Figure: Round elevated vitelliform lesion at the macula in Best's dystrophy

Best's vitelliform dystrophy is characterised by:


 AD inheritence with incomplete penetrance
 abnormal EOG (even in carriers)
 normal ERG
 pre-vitelliform stage: retina normal, EOG abnormal
 vitelliform: cyst-like yellow-orange lesion (age 4-10)
 pseudo-hypopyon: lesion becomes absorbed
 vitelli-eruptive (scrambled egg): reduced vision
 vision usually reasonable until age 40s
 complications: CNVM, geographic atrophy

Regarding congenital lid abnormalities, which is FALSE:


epicanthus inversus is broader below than above
neither congenital entropion nor congenital districhiasis typically cause significant keratopathy
both congenital ectropion and entropion involve the lower lid more frequently than the upper lid
telecanthus implies hypertelorism

Your answer was CORRECT


Explanation

Figure: a. Epicanthus tarsalis; b. epicanthus inversus; c. epicanthus palpebralis; d. epicanthus


supraciliaris

There are four varieties of epicanthus:


 Epicanthus tarsalis: which is the normal racial structure of the eyelids in some Asian patients.
The fold arises in the region of the upper tarsal plate and terminates in the skin of the medial
canthus. The fold is more prominent in the upper lid than the lower lid.
 Epicanthus inversus: is rare and associated with congenital eyelid anomalies. The fold begins
in the lower eyelid tarsal region and extends up to the medial canthus. The fold is broader
below, in the lower lid, than above.
 Epicanthus palpebralis: is a fold that runs from the upper eyelid tarsal region to the lower
border of the orbit. It may be equally distributed between the upper and lower lids.
 Epicanthus supraciliaris: originates in the region of the eyebrow and terminates over the
lacrimal sac.

Hypertelorism implies an abnormally wide distance between medial orbital walls. Telecanthus is
an abnormally wide distance between medial canthi. Telecanthus may be isolated (primary) or
secondary to hypertelorism.

Which is FALSE regarding Peter's anomaly:

absence of posterior corneal tissue

typical inheritance is autosomal dominant

associated with congenital cardiac defects

can result from a mutation at the PAX6 gene


Your answer was CORRECT
Explanation

Figure: Peter's anomaly

Peter's anomaly is typically sporadic. It is characterised by absence of posterior corneal tissue and
leukoma, ±iris adhesions to leukoma, ±lens-corneal adhesions. Opacification of the cornea leads
to an amblyogenic effect on a developing infant. Peters' plus syndrome includes short
disproportionate stature, developmental delay, dysmorphic facial features, cardiac, genito-urinary,
and central nervous system malformation. These systemic findings are seen in up to 60% of
patients. Peters’ is also associated with many other ocular pathologies including glaucoma,
sclerocornea, corectopia, iris hypoplasia, cataract, ICE syndrome, aniridia, iris coloboma,
persistent fetal vasculature and microcornea.

Juvenile idiopathic arthritis-associated iridocyclitis is most common in:


late-onset pauciarticular disease
early-onset pauciarticular disease
Still's disease
late-onset polyarticular disease

Your answer was CORRECT


Explanation
Risk factors for iridocyclitis in JIA include:
 ANA positive
 rheumatoid factor negative
 pauciarticular type (90% of cases)
 female
 involvement of lower extremity joints
 lack of involvement of the wrists
Systemic JIA with fever and rash is rarely associated with iritis.

The systemic disorder most commonly associated with the blepharophimosis syndrome is:
hypospadias
primary amenorrhoea
coarctation of the aorta
diabetes mellitus
hypothyroidism

Your answer was CORRECT


Explanation

Figure: Blepharophimosis

Note: primary amenorrhea is the most common condition associated with blepharophimosis.

Blepharophimosis is characterised by:


 congenital condition
 autosomal dominant inheritance (known as the Kohn-Ramono syndrome)
 widened intercanthal distance (telecanthus)
 ptosis
 blepharophimosis
 epicanthus inversus
 associated findings:
o lower eyelid ectropion is commonly seen
o poor development of the orbital rims and nasal bones
o hypertelorism
o systemic: primary amenorrhoea

Which clinical feature if present is most helpful in the presumptive diagnosis of retinoblastoma:
ophthalmoscopically evident calcification
strabismus

spontaneous hyphaema

heterochromia iridis

Your answer was CORRECT


Explanation
Two diagnostic features of retinoblastoma are chalky white deposits on the tumour (which
represent calcifications) and multiple clumps of tumour cells floating in the vitreous (vitreous
seeding).

Which drops are most appropriate for cycloplegic refraction in a 6-month-old child?

1.0% cyclopentolate and 10% phenylephrine

0.5% cyclopentolate and 2.5% phenylephrine


1.0% atropine and 2.5% phenylephrine

1.0% cyclopentolate and 2.5% phenylephrine

Your answer was CORRECT


Explanation
For infants less than 12 months, the dilating drops of choice are 0.5% (or 0.25%) cyclopentolate
and 2.5% (or 1%) phenylephrine. Note that atropine drops should be avoided in young infants
because of the potential for anticholinergic adverse effects. For infants older than 1 year,
cyclopentolate 1% with 2.5% phenylephrine are usually employed.

This question came in the 2014 FRCOphth.

Reference: Clinical Strabismus Management: Principles and Surgical Techniques; Arthur L.


Rosenbaum, Alvina Pauline Santiago; 1999.

The most common congenital infection in humans is:


syphilis
rubella
toxoplasmosis
cytomegalovirus
herpes simplex virus

Your answer was CORRECT


Explanation
Approximately 2% of human infants are infected by CMV. Most cases are silent.

Ocular features of congenital CMV infection:


 chorioretinitis (most common feature)
 optic atrophy
 pigmentary retinopathy
 strabismus

Systemic features of congenital CMV:


 hepatosplenomegaly
 pneumonia
 sensorineural hearing impairment
 microcephaly
 seizures

Which of the following causes of ophthalmia neonatorum requires intravenous penicillin for
adequate treatment?

Crede's prophylaxis

HSV

Chlamydiae subtypes D through K

Neisseria gonorrhoeae

Your answer was CORRECT


Explanation
Gonorrhoeael neonatal conjunctivitis may be associated with corneal ulceration and
endophthalmitis. It requires treatment with iv penicillin.

All are true of retinoblastoma EXCEPT:

40% of RB is hereditary

hereditary RB occurs at a younger age and is more likely bilateral

a patient with RB but no family history is unlikely to develop tumours elsewhwere


familial RB is inherited as autosomal dominant with high penetrance

Your answer was CORRECT


Explanation
40% of RB is hereditary. Hereditary occurs at a younger age (mean of 1 year versus 2 years for
non-hereditary), is more likely to be bilateral, and is associated with non-ocular malignancies.
Because a patient has no family history, does NOT mean that they have non-hereditary RB. 25-
30% of hereditary RB cases are non-familial (since the rate of new mutation is high).

Non-hereditary (sporadic) RB refers to the fact that both alleles are normal after fertilisation and
there are 2-hits during early life. 60% of RB is non-hereditary, which is always unilateral, and
there is no increased risk of non-ocular malignancies.

Which condition is more commonly seen in boys than in girls?

congenital oculomotor apraxia

Duane's syndrome

Mobius syndrome

congenital third nerve palsy

congenital sixth nerve palsy

Your answer was CORRECT


Explanation
Congenital oculomotor apraxia is more commonly seen in boys.
Duane's syndrome is more common in girls.
There is no clear gender predilection in Mobius syndrome or in congenital third and sixth nerve
palsies.

What is the treatment for malignant hyperthermia?

nitroglycerin

insulin and potassium

dantrolene

atropine
Your answer was CORRECT
Explanation
Dantrolene is a muscle relaxant, which stabilises cell membranes and prevents the release of
calcium from the sarcoplasmic reticulum of muscle cells. Other important measures in the
management of malignant hyperthermia include discontinuation of the offending agent, cooling of
the patient, hydration, hyperventilation, and bicarbonate for the acidosis.

Which of the following features helps to differentiate congenital hereditary endothelial dystrophy
from congenital hereditary stromal dystrophy:

1. epithelial oedema
2. intraocular pressure
3. corneal thickness

1 only

2 only

1 and 3
1 and 2

2 and 3

Your answer was CORRECT


Explanation
In congenital hereditary stromal dystrophy, the cloudy stroma is of normal thickness, and the
epithelium is normal. In congenital heriditary endothelial dystrophy (CHED), there is epithelial
oedema and a thickened stroma. In both conditions, IOP is usually normal.

Neuroblastoma is characterised by all of the following EXCEPT:


periorbital ecchymosis
metastasis from the adrenal gland
poor prognosis if diagnosed before 1 year of age
possible spontaneous regression

Your answer was CORRECT


Explanation
Neuroblastoma is characterised by the following:
 most frequent orbital metastasis in children
 originate from: adrenals, mediastinum, and neck
 20% exhibit orbital involvement
 mean age of presentation of orbital neuroblastoma metastases: 2 years
 prognosis generally poor but better if present at under 1 year
 spontaneous regression of this tumor may be seen in rare instances

Which is FALSE regarding Peter's anomaly?


half of patients have glaucoma

there is absence of Descemet's membrane and endothelium centrally

it can be associated with cataract

most cases are unilateral

Your answer was CORRECT


Explanation

Figure: Peter's anomaly

Peter's anomaly is bilateral in two-thirds of cases. Other statements are true.

Peter's anomaly is typically sporadic and it is bilateral in two-thirds of cases. It is characterised by


absence of posterior corneal tissue and leukoma, ±iris adhesions to leukoma, ±lens-corneal
adhesions. Opacification of the cornea leads to an amblyogenic effect on a developing infant.
Peters' plus syndrome includes short disproportionate stature, developmental delay, dysmorphic
facial features, cardiac, genito-urinary, and central nervous system malformation. These systemic
findings are seen in up to 60% of patients. Peters’ is also associated with many other ocular
pathologies including glaucoma, sclerocornea, corectopia, iris hypoplasia, cataract, ICE syndrome,
aniridia, iris coloboma, persistent fetal vasculature and microcornea.
The primary abnormality in simple congenital ptosis relates to the:

levator muscle

levator innervation

levator aponeurosis

third nerve nucleus

Your answer was CORRECT


Explanation

Figure: Congenital ptosis

Congenital ptosis is typically myogenic. It is caused by maldevelopment isolated to the levator-


superior rectus complex.

An 11-year-old girl with a history of arthralgia develops a chronic, bilateral anterior chamber
inflammation with flare and no cells.

Which statement is most likely TRUE?


this patient is antinuclear antibody (ANA) negative
this patient has pauciarticular disease involving the hands and wrists
this patient is rheumatoid factor positive
this patient has pauciarticular disease involving the lower extremities without wrist
involvement

Your answer was CORRECT


Explanation
This patient has juvenile idiopathic arthritis (JIA), which presents in children younger than 16
years of age.
Risk factors for ocular involvement include:
 ANA positive
 rheumatoid factor negative
 pauciarticular type (90% of cases)
 female
 involvement of lower extremity joints
 lack of involvement of the wrists
Systemic JIA with fever and rash is rarely associated with iritis.

A 5-year-old asymptomatic boy has bilateral superficial corneal vascularisation inferiorly. What is
the most likely cause?

allergy

previous adenoviral disease

blepharitis
rosacea

Your answer was CORRECT


Explanation
The most likely cause of asymptomatic inferior corneal vascularisation is childhood
blepharokeratoconjunctivitis (BKC). While BKC may be associated with dermatological
conditions such as rosacea and atopy, these are less common than in adult disease. Allergic eye
disease, including vernal keratoconjunctivitis are more likely to produce superior corneal changes.

This question appeared in the 2014 FRCOphth Part 2.

For congenital nasolacrimal duct obstructing persisting to 12 months of age, an initial probing
with irrigation is successful in what proportion of children?

60%
75%

90%
33%

Your answer was CORRECT


Explanation
Probing and syringing is successful over 90% of the time. Failure is generally followed by a repeat
probing and irrigation. If this fails a second time, then silastic intubation is tried. If this also fails,
then dacryocystorhinostomy is the next step.
A 9-year-old girl has recurrent spontaneous hyphaemas and multiple yellow papules on her skin.

What is the most likely histological finding?

Touton cells
Schaumann

Langerhan

Cowdry type A bodies

Langhan

Your answer was CORRECT


Explanation

Figure: Juvenile xanthogranuloma of iris.

The patient in the question above has a history suggestive of juvenile xanthogranuloma (JXG),
which can affect the iris causing recurrent hyphaema and also the skin causing yellow-orange
papules. JXG is characterised histologically by Touton cells. Touton cells are formed by fusion of
epithelioid cells (macrophages) with nuclei surrounded by foamy cytoplasm.

A 2-year-old boy presents with periorbital ecchymosis. The differential diagnosis includes all of
the following EXCEPT:

leukaemia

lymphangioma

dermoid cyst

neuroblastoma
Your answer was CORRECT
Explanation
Dermoid cysts may rupture and spill their contents, inciting a significant amount of inflammation,
but bleeding does not occur.

Neuroblastoma is the most common source of orbital metastasis in children. Metastatic


neuroblastoma produces proptosis with periorbital ecchymosis. Bilateral involvement is seen in
half of the cases.

Orbital infiltration by leukaemia causes lid swelling, proptosis, and ecchymosis.

Lymphangioma lesions consist of lymph-filled channels lined by endothelium and separated by


thin, delicate walls with small blood vessels that are broken easily. Lymphangiomas infiltrate
orbital tissues extensively, and intralesional bleeding is common.

Paediatric penetrating keratoplasty surgery may be complicated by all of the following EXCEPT:

amblyopia

slow or delayed healing


glaucoma

graft rejection

Your answer was CORRECT


Explanation
Corneal transplants in children and young adults heal very quickly with sutures becoming
vascularized and loosening within the first several months. Other options are true.

Which of the following is generally associated with a conjunctivitis developing 5 to 14 days


following birth?

Neisseria gonorrhoeae

Herpes simplex virus

Crede's prophylaxis

Adenovirus

Your answer was CORRECT


Explanation
The time of onset of neonatal conjunctivitis is important in determining aetiology. Silver nitrate
prophylaxis (medicamentosa) tends to present earliest (within 24 hours of birth), followed by
gonococcus (2 days), herpes simplex virus (5 to 14 days), and chlamydiae (1-3 weeks)

The anterior segment dysgeneses reflect embryonic abnormalities related to which cell line:
surface ectoderm
neuroectoderm
endoderm
neural crest
mesoderm

Your answer was CORRECT


Explanation
The anterior segment dysgeneses are believed to reflect abnormalities in development of tissues of
neural crest origin.

Embryological origins of the eye are as follows: 1

 Neuroectoderm
o retina - neurosensory and RPE
o epithelial lining of iris and ciliary body
o optic nerves
 Surface ectoderm
o lens
o corneal epithelium
o conjunctival epithelium
o lacrimal gland
o nasolacrimal system
o Meibomian glands
 Neural crest
o sclera
o iris stroma
o cornea: stroma and endothelium
o TM and Schlemm's canal
o extraocular muscle
o ciliary muscle
o connective tissue and bony structure of the orbit
 Mesoderm
o extraocular muscles
o endothelial lining of blood vessels of the eye
o blood vessels in sclera and choroid
o sclera
o vitreous
o suspensory fibres
o angle outflow apparatus
1. Review of Ophthalmology by Friedman et al

In a child diagnosed with retinoblastoma, where are you MOST likely to find a genetic
abnormality:

chromosome 11p

chromosome 15

chromosome 13q14

chromosome 1p

Your answer was CORRECT


Explanation
Some popular genetic correlations are listed below:

ChromosomeCondition

13q14 retinoblastoma

1p Stargardt's

19 myotonic dystrophy

granular, Avellino, Reis‐Buckler and


5q
Lattice I dystrophies

15 Marfans

11p aniridia

20 Alagille syndrome

HLAB51 Behcet's disease

HLA A29 Birdshot chorioretinopathy

All of the following are craniofacial synostosis syndromes EXCEPT:

Apert's

Treacher-Collins
Pfeiffer's

Crouzon's

Your answer was CORRECT


Explanation

Figure: Treacher Collins syndrome

Crouzon's, Apert's and Pfeiffer's are all categorised as craniofacial synostosis syndromes. These
conditions are the result of premature closure of one or more cranial sutures. Treacher-Collins
syndrome, on the other hand, is categorised as a type of clefting syndrome. These patients
typically have hypoplasia of the mid-face, pseudocolobomas of the eyelids, downward angle to the
lateral canthi, and dental and ear anomalies

Which phacomatosis has NO known mode of inheritance?

Sturge-Weber syndrome

Neurofibromatosis

Tuberous sclerosis

von Hippel-Lindau disease

Your answer was CORRECT


Explanation
Sturge-Weber and Wyburn-Mason syndromes have no known mode of inheritance. Other
phacomatoses tend to be autosomal dominant including tuberous sclerosis, neurofibromatosis and
von Hippel-Lindau.

Which one of the following statements regarding megalocornea is FALSE?

tearing and IOP are important factors in the work-up

it is often associated with anterior megalophthalmos, an X-linked recessive disorder

the simple form of megalocornea is usually seen as a bilateral condition

in infants older than 2 years, the condition is defined as a clear, normal-appearing cornea
with a diameter measuring greater than 12 mm
Your answer was CORRECT
Explanation
By 2 years of age, the cornea is approximately adult size. Simple megalocornea is defined as both
corneas measuring greater than 13 mm in children older than 2 years, and greater than 12 mm in
infants. Congenital glaucoma is associated with epiphora and increased IOP and must be ruled out
of the differential diagnosis. The most common type of megalocornea is associated with anterior
megalophthalmos, an X-linked recessive disorder.

All are true of Coats' disease EXCEPT:

older children have a more benign disease course

the condition is usually bilateral


telangectasis is a recognised feature and is most common in the temporal periphery

there is a bimodal age presentation

rubeosis is a recognised complication

Your answer was CORRECT


Explanation
Figure: FFA of Coats' showing characteristic miliary aneurysms and telangiectatic vessels with
exudation

In regards the question above, Coats' is usually unilateral; while other options supplied are true.

Which statement regarding the treatment of retinoblastoma is FALSE?


for large tumours with poor visual potential enucleation is appropriate

photocoagulation may be useful for small, obviously localised tumours

cryotherapy is avoided because it typically leads to dissemination of viable tumour cells


within the eye
both plaque brachytherapy and external beam irradiation may be effective

in metastatic cases, chemotherapy is used

Your answer was CORRECT


Explanation
Cryotherapy is as effective as photocoagulation for small tumours that are difficult to manage with
photocoagulation.

Which of the following is NOT a typical feature of congenital toxoplasmosis:


deafness
diffuse pigmentary retinopathy
seizures with intracranial calcifications
hepatosplenomegaly

Your answer was CORRECT


Explanation
Focal chorioretinal lesions are common but diffuse retinopathy is not seen in congenital
toxoplasmosis.

Features of congenital toxoplasmosis in infancy and later in life:*


 Abnormal spinal fluid
 Anaemia
 Chorioretinitis †
 Seizures
 Deafness
 Fever
 Growth retardation
 Hepatomegaly
 Hydrocephalus †
 Intracranial calcifications †
 Jaundice
 Learning disabilities
 Lymphadenopathy
 Maculopapular rash
 Mental retardation
 Microcephaly
 Spasticity and palsies
 Splenomegaly
 Thrombocytopenia
 Microphthalmia
 Cataract

* Most neonates with congenital toxoplasmosis are asymptomatic as determined by routine


newborn examination.

† Sign in the classic triad suggesting the presence of congenital toxoplasmosis.

A 7-year-old boy presents with acute left blurred vision. On examination, there is a left hypopyon,
an inferior retinal granuloma with pars planitis. The right eye is normal. There is no past ocular or
medical history.

What is the most likely diagnosis?

Coat disease

Retinoblastoma

Toxoplasmosis

Toxocariasis
Leukaemia

Your answer was CORRECT


Explanation
Figure: ocular toxocariasis with retinal granuloma and vitreoretinal band

Regarding the question above, a retinal granuloma is a fairly rare and specific finding and in the
context of panuveitis is suggestive of Toxocariasis.

Toxocara may present with an inflamed or quiet eye. In younger children, it tends to present with
a chronic endophthalmitis, while in older children a quiet posterior or peripheral granuloma with
or without an associated tractional vitreoretinal band may be present. Treatment involves an anti-
helminthic - and systemic steroids if ocular inflammation is present.

All are important in the management of malignant hyperthermia EXCEPT:


dantrolene
hypoventilation
discontinuing halothane anaesthetic
hydration including bicarbonate supplementation

Your answer was CORRECT


Explanation
Dantrolene is a muscle relaxant, which stabilises cell membranes and prevents the release of
calcium from the sarcoplasmic reticulum of muscle cells. It is useful in the treatment of malignant
hyperthermia. Other important measures in the management of malignant hyperthermia include
discontinuation of the offending agent, cooling of the patient, hydration, hyperventilation, and
bicarbonate for the acidosis.

Malignant hyperthermia is characterised by:


 incidence of between 1: 6,000 and 1: 30,000
 more common in children
 triggered by succinylcholine, halothane, enflurane, and isoflurane
 mortality rate now less than 10%
 creatine phosphokinase is elevated in up to two-thirds of patients
 the earliest sign is tachycardia or elevated end-tidal carbon dioxide
 other early signs:
o unstable blood pressure
o tachypnea
o sweating
o muscle rigidity
o cyanosis
o dark urine
 a rise in temperature is a late sign

Congenital lid colobomas are most characteristic of which systemic syndrome?


Hallermann-Streiff syndrome
Pierre Robin syndrome
Goldenhar syndrome
Stickler syndrome

Your answer was CORRECT


Explanation

Goldenhar syndrome is characterised by:


 preauricular appendages
 aural fistulas
 maxillary or mandibular hypoplasia
 hemifacial microsomia
 vertebral deformities
 lid coloboma
 Duane's syndrome
 limbal dermoids
 sensorineural deafness
An 18-month old girl with a history of fetal alcohol syndrome has been referred to clinic with
concerns regarding her visual development.

Findings on examination might include all of the following EXCEPT:


optic nerve hypoplasia
microphthalmia
Peters' anomaly
sclerocornea
tortuous retinal vessels

Your answer was CORRECT


Explanation

Figure: facial features of fetal alcohol syndrome

Ocular features of fetal alcohol syndrome:


 epicanthal folds
 strabismus
 blepharophimosis
 long eyelashes
 microphthalmia
 telecanthus
 Peter's anomaly
 anterior segment dysgenesis
 tortuous retinal vessels
 optic nerve hypoplasia
 persistent hyaloid vessel

Which is FALSE regarding retinoblastoma?


increased incidence of secondary malignant tumors later in life
the Reese-Ellsworth classification provides prognostic information about survival
can present with orbital cellulitis, strabismus, or hyphaema
two mutations in chromosome 13 are required to develop retinoblastoma
Your answer was CORRECT
Explanation
The Reese-Ellsworth classification was developed in the 1950s as a guide for predicting visual
prognosis in eyes treated by methods other than enucleation. It has been used erroneously to
predict patient prognosis for life.

Retinoblastoma is characterised by the following:


 the most common ocular malignant tumor of childhood
 leukocoria most common presentation
 other features: strabismus, uveitis, cellulitis, angle closure, hyphema
 mutation in the long arm of chromosome 13 (13q14)
 two-thirds are unilateral and non-hereditary
 autosomal dominant pattern with high penetrance for:
o all patients with familial disease
o all non-familial bilaterally affected patients
o 15% of non-familial unilaterally affected patients
 hereditary form is associated with malignancies later in life, especially sarcomas

In which of the following causes of neonatal conjunctivitis can the diagnosis be made by Giemsa
stain alone?

HSV
Crede's prophylaxis

Neisseria gonorrhoeae

Chlamydiae subtypes D through K

Your answer was CORRECT


Explanation
Chlamydia stains with Giemsa.

The retinal findings in shaken-baby syndrome are LEAST like:

Terson's syndrome

Purtscher's retinopathy

commotio retinae

central retinal vein occlusion

Your answer was CORRECT


Explanation
Retinal hemorrhages are the cardinal manifestation of shaken-baby syndrome. The incidence of
retinal hemorrhages in SBS is approximately 85%. Classically, children with SBS have retinal
hemorrhages which are multilayered - preretinal, intraretinal, and subretinal. They are usually too
numerous to count and extend out to the retinal periphery (i.e. not just confined to the posterior
pole). Terson's, Purtscher's and central retinal vein occlusion will have similar haemorhagic
appearances, unlike commotio retinae where there is retinal swelling from contra-coup forces after
blunt trauma but no haemorhages.

This question appeared in the 2014 FRCOphth Part 2.

A young child has optic disc drusen, posterior embryotoxon, jaundice and tetralogy of Fallot.

What is the most likely diagnosis?


Cornelia de Lange syndrome
Aicardi syndrome
Allagille syndrome
Hallerman Streiff syndrome
CHARGE syndrome

Your answer was CORRECT


Explanation
Allagille syndrome is characterised by:
 autosomal dominant condition
 gene deletion disrupts developmental pathway
 optic disc drusen
 posterior embryotoxon
 jaundice from bile duct atresia
 tetralogy of Fallot

A mother brings her 12-month-old child to you because of chronic right-sided epiphora. She has
been massaging the right nasolacrimal sac for the past 6 months; however, the epiphora has
persisted.

The next most suitable therapeutic recommendation would be:

maxitrol ointment BD for 2 months

nasolacrimal probing
silicon intubation

DCR

ongoing massage
Your answer was CORRECT
Explanation
Congenital nasolacrimal system obstruction is relatively common and is seen in 2% to 4% of
newborn infants. In 80% to 90% of patients, the membranous valve of Hasner opens
spontaneously or with gentle massage of the nasolacrimal sac within the first 6 to 12 months of
life. After 12 months, the chances of spontaneously opening are less and probing is indicated.
Should multiple probings fail, silicon tube intubation can be tried, and should this fail DCR may
then rarely be considered.

What procedure is appropriate for a 3-year-old child with congenital ptosis and levator function of
3mm?

aponeurosis advancement

levator resection

brow suspension with autologous fascia lata

brow suspension with Mersilene mesh

Your answer was CORRECT


Explanation
Poor levator function (less than 4mm) requires a brow suspension. Autologous fascia lata is
generally the preferred technique for brow suspension but it is usually necessary to wait until the
age of about 5 years for a child's leg to be sufficiently developed to harvest fascia lata. There are a
variety of non-autogenous suspensory materials that can be used (including Mersilene) but all
synthetic materials have disadvantages such as: infection, intrusion, breakage, or foreign body
granuloma formation.

Which of the following causes of neonatal conjunctivitis, if untreated, may progress to otitis media
and pneumonitis?

Chlamydia
HSV

Crede's prophylaxis

Neisseria gonorrhoeae

Your answer was CORRECT


Explanation
Chlamydia may be associated with otitis media and pneumonitis.
A 9-year-old boy has bilateral partial hearing loss, abnormal renal function and visual impairment.
Slit-lamp examination reveals posterior polymorphous dystrophy and peripheral retinal flecks. His
older brother has the same condition.

What is the most likely diagnosis?


Bietti crystalline dystrophy
Fabry's disease
Alport's syndrome
Lowe's syndrome

Your answer was CORRECT


Explanation

Figure: slit lamp of posterior polymorphous corneal dystrophy.

The clinical description in the question above is in keeping with Alport's syndrome.

Alport's syndrome is characterised by:


 X-linked recessive (80%) and X-linked dominant (5%) inheritance patterns
 defect in Type IV basement membrane collagen
 renal failure secondary to glomerular dysfunction
 megalocornea
 posterior polymorphous dystrophy
 anterior lenticonus
 anterior sub-capsular cataract
 corneal arcus
 peripheral retinal flecks

Which statement about juvenile idiopathic arthritis is FALSE?


females are more at risk of uveitis

it is the most common cause of anterior uveitis in the paediatric population

the age of onset is typically under 2 years


the uveitis of JIA is nearly always entirely anterior

Your answer was CORRECT


Explanation
Uveitis is rare under the age of 2 years. Other options are true.

The organism most commonly responsible for preseptal cellulitis in children is:

Staphylococus aureus
Streptococcus pyogenes

Pseudomonas aeruginosa

Haemophilus influenzae

Your answer was CORRECT


Explanation
The most common cause of pre-septal cellulitis in children is minor skin trauma and the most
common organism is Staph aureus.

To test the visual acuity of an 18-month-old child, which of the following is most appropriate?

Crowded Kays

Sheridan-Gardener

Sonksen

Cardiff cards

Your answer was CORRECT


Explanation
Cardiff preferential looking cards are best for children under the age of 2. Picture matching is
appropriate over the age of 2, using tests such as the Kays. Over the age of 3, letter matching of
single optotypes (e.g. Sheridan-Gardener) or crowded optotypes (Sonksen) may be used.

This question came in the 2014 FRCOphth.


Goldenhar's syndrome is NOT associated with:
hypertelorism
Duane's syndrome
vertebral anomalies
eyelid colobomas

Your answer was CORRECT


Explanation

Figure: Child with Goldenhar syndrome.

Goldenhar's syndrome is characterised by:


 preauricular appendages
 aural fistulas
 maxillary or mandibular hypoplasia
 hemifacial microsomia
 vertebral deformities (e.g. scoliosis)
 notching of the upper eyelid
 Duane's syndrome
 limbal dermoids
 sensorineural deafness

A 10-year-old girl has insiduous-onset blurred vision to 6/15 right eye, 6/12 left eye. Examination
reveals pale-yellow pisciform plaques varying in size from 200-600 micrometres at the level of the
RPE bilaterally throughout both posterior poles.

Given the most likely condition, which one of the following would be most useful in establishing
the diagnosis?
EOG

VEP

Fluorescein angiogram
ERG

Your answer was CORRECT


Explanation
This is a patient with Stargardt's disease. The majority of patients have normal electrophysiologic
tests, although ERG amplitudes may be in the low-normal range. FFA in Stargardt's shows a
typical dark-chroid appearance, which is useful to confirm the diagnosis.

If the parents of a child with a congenital eye defect were normal but found to be first cousins,
what is the most likely mode of inheritance:

mitochondrial inheritance

autosomal dominant

autosomal recessive
X-linked recessive

Your answer was CORRECT


Explanation
When parents are phenotypically normal but produce an affected child it implies a recessive gene.
Consanguinity increases the risk of autosomal recessive disorders because of the smaller gene
pool creating more likelihood of homozygous gene loci in the offspring.

In myogenic congenital ptosis, the levator complex in the ptotic eye is:
disinserted from the tarsus
innervated by cranial nerve VII
histologically abnormal with decreased muscle fibers and fatty infiltrates
absent below Whitnall's ligament

Your answer was CORRECT


Explanation
Figure: Myogenic congenital ptosis

Myogenic congenital ptosis is characterised by:


 dysgenesis of the levator muscle complex
 the levator complex is atrophic with replacement of muscle fibers by fibrous or adipose tissue

What is the main concern in an upper lid capillary haemangioma in a 3-month old infant?
metastasis
spontaneous haemorrhage
optic nerve compression
orbital cellulitis
amblyopia

Your answer was CORRECT


Explanation
Upper lid capillary haemangiomas may be large enough to block the visual axis and cause
deprivation amblyopia.

Characteristics of capillary haemangioma:


 appear a short time after birth
 composed of blood vessels that have an abnormal growth proliferation
 can be superficial or deep in the orbit
 capillary haemangiomas are characteristically high flow lesions
 majority: spontaneous involution
 70% will resolve before 7 years of age

A child has vortex keratopathy, wedge-shaped cataract, and retinal vessel tortuosity. He is under
the paediatricians for investigation of peripheral neuropathy, renal impairment and a widespread,
red spot-like rash.

What is the most likely diagnosis?


Fabry disease
Lowe syndrome
Allagille syndrome
Alport syndrome
Aicardi syndrome

Your answer was CORRECT


Explanation

Figure: angiokeratomas typical of Fabry disease

Fabry disease is characterised by:


 X-linked recessive inheritance
 Deficiency in alpha-galactosidase enzyme
 Peripheral neuropathy
 GI symptoms
 Cardiac and renal impairment
 Skin: widespread angiokeratomas
 Ocular:
o vortex keratopathy
o cataract
o retinal vessel tortuosity

The most common cause of combined deafness and blindness in childhood is:

Cogan's syndrome

congenital syphilis
Usher's syndrome
congenital rubella

Vogt-Koyanagi-Harada syndrome

Your answer was CORRECT


Explanation
Usher's syndrome accounts for half of all cases of combined deafness and blindness in childhood
with pigmentary retinopathy developing before puberty.

Which is FALSE regarding congenital hereditary endothelial dystrophy?


there are usually no associated systemic abnormalities
nystagmus is absent in the recessive form of the disease
there are usually no associated congenital ocular abnormalities
corneal clouding is absent at birth in the dominant form of the disease
the recessive form is non-progressive, whereas the dominantly inherited form is slowly
progressive

Your answer was CORRECT


Explanation
Figure: congenital hereditary endothelial dystrophy

CHED has two forms:


 Recessive inheritance
o corneal oedema present at birth
o non-progressive
o nystagmus
 Dominant inheritance
o decompensation occurs in the first or second year of life
o progressive
o nystagmus is NOT a feature
There are no known ocular or systemic associations of CHED.

Which of the following causes of neonatal conjunctivitis, if left untreated, may progress to
meningitis?

Crede's prophylaxis

Neisseria gonorrhoeae

Chlamydiae subtypes D through K

Herpes simplex virus

Your answer was CORRECT


Explanation
HSV neonatal conjunctivitis may progress to meningitis and is treated with iv acyclovir.

Which of the following statements regarding paediatric uveitis is TRUE:

B27-associated uveitis is the most prevalent aetiology of uveitis in children

Fuch's uveitis does not occur before the age of 12 years

the systemic form of juvenile idiopathic arthritis can lead to severe vision-threatening uveitis

paediatric uveitis represents 10% of all uveitis cases

Your answer was CORRECT


Explanation
JIA is considered the most common cause of uveitis in children. The systemic form of JIA almost
never produces uveitis. Fuch's heterochromic iridocyclitis can occur in the paediatric population.

Which of the following statements regarding paediatric uveitis is TRUE:

B27-associated uveitis is the most prevalent aetiology of uveitis in children

Fuch's uveitis does not occur before the age of 12 years

the systemic form of juvenile idiopathic arthritis can lead to severe vision-threatening uveitis

paediatric uveitis represents 10% of all uveitis cases

Your answer was CORRECT


Explanation
JIA is considered the most common cause of uveitis in children. The systemic form of JIA almost
never produces uveitis. Fuch's heterochromic iridocyclitis can occur in the paediatric population.

Which statement is TRUE regarding congenital hereditary endothelial dystrophy (CHED)?

it is primarily a unilateral disease

the dominant form is more likely to demonstrate nystagmus than the recessive form

it can be differentiated from congenital hereditary stromal dystrophy by pachymetry

the dominant form is stationary while the recessive form is progressive

Your answer was CORRECT


Explanation
CHED is one of many diseases that causes cloudy corneas in children. It is a bilateral disease and
comes in two forms, both having markedly thickened corneas. The autosomal dominant form is
progressive and presents between 1 and 2 years of age with photophobia and tearing. Nystagmus
is usually absent. The autosomal recessive form is present since birth and stationary. A sensory
nystagmus is more commonly associated with the recessive form because vision is poor since
birth. This disease must be differentiated from congenital glaucoma, which may present with
buphthalmos and enlarged corneal diameters.

CHSD (stromal dystrophy) usually presents with only central corneal involvement and a clear
periphery. In CHSD, only the stroma is involved; there is no corneal oedema, photophobia, or
tearing. CHSD and CHED can also be differentiated on the basis of pachymetry; CHSD has
normal corneal thickness.

The therapeutic management of JIA-associated uveitis includes all EXCEPT:

infliximab

etanercept
methotrexate

cycloplegics

topical steroids

Your answer was CORRECT


Explanation
Methotrexate is typically the first-line systemic immunosuppressive treatment for patients with
JIA, and this often controls the uveitis. Anti-TNF agents have shown some benefit in resistant
cases, but in this class of drugs infliximab has produced better results than etanercept, which has
been shown to have a minimal effect.

A 3-year-old presents with periocular ecchymosis, sub-conjunctival haemorrhage and proptosis.


There is no history of trauma. The CT scan shows a mass within the lateral orbital wall. There is
no bone erosion.

What is the most likely diagnosis?


neuroblastoma
orbital cellulitis
lacrimal gland tumour
spontaneous orbital haemorrhage
embryonal sarcoma

Your answer was CORRECT


Explanation

Figure: Right proptosis and ptosis due to metastatic neuroblastoma

Neuroblastoma is characterised by the following:


 most common extracranial solid tumour in children
 originates from: adrenals, mediastinum, and neck
 20% exhibit orbital involvement
 most frequent orbital metastasis in children
 mean age of presentation of orbital neuroblastoma metastases: 2 years
 orbital symptoms: proptosis and periorbital ecchymosis (raccoon eyes)
 prognosis generally poor but better if it presents under 1 year of age
 spontaneous regression of this tumor may be seen in rare instances

Note: rhabdomyosarcomas typically erode bone and median age at presentation is 8-9 years

A 14-year-old girl is struggling to read homework and complains of poor central vision. Her
acuities are 6/24 bilaterally. The first year ophthalmology trainee reports no abnormalities on
dilated fundus examination.

What is the most likely diagnosis?


enhanced S-cone syndrome
Best's disease
central serous chorioretinopathy
Sjogren-Larsson syndrome
Stargardt's disease

Your answer was CORRECT


Explanation
The age of presentation and normal fundus appearance suggest Stargardt's disease.

Stargardt’s (fundus flavimaculatus) is characterised by:


 most common inherited macular dystrophy
 autosomal recessive
 presents in first 2 decades
 bilateral reduced vision
 fundus may appear normal
 early: mottling of RPE; pisciform flecks at the level of the RPE at the posterior pole
 later: aggregation of flecks to form an atrophic patch and bull's eye appearance
 FFA: dark or 'silent choroid' due to blockage of the background choroidal fluorescence by
deposition of lipofuscin
 visual prognosis is generally poor with vision stablising to 6/60

Which of the following causes of neonatal conjunctivitis, if untreated, may progress to corneal
ulceration and endophthalmitis?

HSV

Crede's prophylaxis

Neisseria gonorrhoeae

Chlamydiae subtypes D through K

Your answer was CORRECT


Explanation
Gonorrhoeae may be associated with corneal ulceration and endophthalmitis. Gonorrhoeae
neonatal conjunctivitis requires i.v. antibiotics.

All of the following are characteristics of a childhood eyelid capillary haemangioma EXCEPT:
spontaneous regression in most cases
low flow lesion angiographically
similar lesions can be found deep in the orbit
usually presents within the first 6 months of age

Your answer was CORRECT


Explanation
Capillary haemangiomas are high-flow lesions, in contrast to cavernous hemangiomas, which are
haemodynamically low-flow.

Characteristics of capillary haemangioma:


 appear a short time after birth
 composed of blood vessels that have an abnormal growth proliferation
 can be superficial or deep in the orbit
 capillary haemangiomas are characteristically high flow lesions
 majority: spontaneous involution
 70% will resolve before 7 years of age

Breaks in Descemet's membrane associated with birth trauma are typically:


petalloid

horizontal

diagonal

tigroid

vertical

Your answer was CORRECT


Explanation

Figure: birth trauma

In both birth trauma and congenital glaucoma corneal oedema and associated breaks in Descemet's
membrane may be present. However, the IOP is elevated in congenital glaucoma while it is
normal in trauma. Additionally, the Descemet's breaks are classically horizontal in glaucoma
(Haab's striae) as opposed to vertical in trauma.

All of the following are potentially important complications of eyelid capillary haemangiomas in
childhood EXCEPT:
cosmetic deformity
occlusion amblyopia
significant astigmatism
ocular invasion
Your answer was CORRECT
Explanation
Ocular invasion is not a feature of capillary haemangioma.

Indications for treatment of eyelid capillary hemangiomas in childhood include:


 significant astigmatism
 significant cosmetic deformity
 significant occlusion

The most common juvenile-onset macular dystrophy is:

Best's disease

North Carolina macular dystrophy

Stargardt's disease

Familial dominant drusen

Your answer was CORRECT


Explanation
Figure: Stargardt's disease showing macula RPE changes and pisciform flecks in the mid-
periphery from lipofuscin deposition (fundus flavimaculatus)

Stargardt's disease is the most common juvenile-onset macular dystrophy.

Sensory nystagmus in an infant may arise in all of the following conditions EXCEPT:

aniridia

bilateral optic nerve hypoplasia

rod monochromatism
infantile esotropia

Your answer was CORRECT


Explanation
Infantile esotropia is associated with manifest-latent nystagmus. Sensory nystagmus, which is
typically a pendular nystagmus, occurs due to bilateral visual deprivation from severe media
opacities or retinal/foveal dysgenesis.

All of the following are associated with blepharophimosis syndrome EXCEPT:


blepharoptosis
epicanthus inversus
hypertelorism
telecanthus

Your answer was CORRECT


Explanation

Blepharophimosis syndrome is characterised by:


 blepharophimosis
 blepharoptosis
 epicanthus inversus
 telecanthus
 ectropion in some cases
 euryblepharon in some cases
Hypertelorism is not a feature of this disorder.

As a general point: hypertelorism implies an abnormally wide distance between medial orbital
walls. Telecanthus is an abnormally wide distance between medial canthi, though the
interpupillary distance may be normal. Telecanthus may be isolated (primary) or secondary to
hypertelorism.
A 4-year-old boy presents with a limbal dermoid. Which is FALSE?
an opaque arc in advance of the lesion is typical and is composed of lipid
histological findings include hair follicles and sweat glands
the lesion is anterior to Bowman's membrane allowing removal by simple excision
associations include preauricular skin tags, upper eyelid coloboma, and vertebral anomalies

Your answer was CORRECT


Explanation
Limbal dermoids are:
 choristomatous tissue
 consist of fibrofatty tissue
 may contain hair follicles, sebaceous glands, and sweat glands
 typically located at inferotemporal limbus
 associated with lipid in the corneal stroma surrounding the edge
 can cause: astigmatism and amblyopia
 may involve corneal stroma - post op scarring or even perforation
 associated with Goldenhar's syndrome, which includes:
o preauricular appendages
o aural fistulas
o maxillary or mandibular hypoplasia
o hemifacial microsomia
o vertebral deformities
o notching of the upper eyelid
o Duane's syndrome

A 4-year-old boy presents with a limbal dermoid. Which is FALSE?


an opaque arc in advance of the lesion is typical and is composed of lipid
histological findings include hair follicles and sweat glands
the lesion is anterior to Bowman's membrane allowing removal by simple excision
associations include preauricular skin tags, upper eyelid coloboma, and vertebral anomalies

Your answer was CORRECT


Explanation
Limbal dermoids are:
 choristomatous tissue
 consist of fibrofatty tissue
 may contain hair follicles, sebaceous glands, and sweat glands
 typically located at inferotemporal limbus
 associated with lipid in the corneal stroma surrounding the edge
 can cause: astigmatism and amblyopia
 may involve corneal stroma - post op scarring or even perforation
 associated with Goldenhar's syndrome, which includes:
o preauricular appendages
o aural fistulas
o maxillary or mandibular hypoplasia
o hemifacial microsomia
o vertebral deformities
o notching of the upper eyelid
o Duane's syndrome

A 5-year-old child presents with a vesicular rash affecting the right upper lid and around the
mouth. The right eye is injected but not photophobic.

What is the most likely diagnosis?

toxic epidermal necrolysis

herpes zoster virus

ocular pemphigoid

cytomegalovirus

herpes simplex virus

Your answer was CORRECT


Explanation
A vesicular rash in a child is most likely to be related to Herpes simplex virus or Herpes zoster
virus (in the form of chicken pox rather than shingles). Chicken pox tends to cause a wider
distribution of vesicles, involving the trunk and limbs, making HSV more likely in this case.

This question came in the FRCS (Glasgow) Part 2 in October 2014.

For a healthy 18-month-old toddler, which is the most appropriate test of visual acuity:

Kay pictures

forced preferential looking

Sheridan-Gardner chart

Cardiff acuity cards

Your answer was CORRECT


Explanation
Less than 1 year, forced preferential looking cards such as Keeler are most appropriate

Between 1-year-old to 2-year-old Cardiff acuity cards are suitable.

Kay pictures are used for children with sufficient language skills to name the pictures (usually age
2).

Sheridan-Gardiner or alternatives such as the Sonksen (cards each containing a single letter or
crowded letters) is used for children age 3 or older.

All are true of Coats' disease EXCEPT:

telangectasis is a recognised feature and is most common in the temporal periphery

appearances can be similar to pigmentary retinopathy

presentation occurs mainly in the first decade

male preponderance

older children have a more aggressive disease course

Your answer was CORRECT


Explanation
Older children and adolescents presenting with Coats' tend to have a more benign course, with less
likelihood of progessive exudation and retinal detachment. Other options are true.

Regarding congenital viral infections, which is FALSE:


most cases of congenital herpes simplex infection are caused by maternal viraemia during
gestation

congenital CMV infection is frequently asymptomatic

most cases of congenital CMV infection are caused by maternal viraemia during gestation

congenital HSV is usually symptomatic

Your answer was CORRECT


Explanation
Most neonatal HSV is contracted at the time of delivery caused by passage through an infected
birth canal. By contrast, most neonatal CMV is caused by maternal viraemia during, or shortly
before, pregnancy and passage of the virus from the mother to the foetus across the placenta.
Unlike congenital CMV, which is mostly asymptomatic, congenital HSV infection is usually
symptomatic with vesicular dermatitis, keratoconjunctivitis, pneumonitis, or meningoencephalitis.

All of the following are histopathological sub-types of rhabdomyosarcoma EXCEPT:


embryonal
morpheaform
alveolar
pleomorphic

Your answer was CORRECT


Explanation
Rhabdomyosarcoma sub-types:
 embryonal: most common type
o botryoid
o spindle cell
 alveolar: worst prognosis
 pleomorphic: best prognosis

Note: Morpheaform is a sub-type of basal cell carcinoma.

In Best disease there is massive accumulation of which substance in the RPE cells:

lipofuscin
melanin

myelin

lipid

drusen

Your answer was CORRECT


Explanation
Best disease is a hereditary autosomal dominant macular degeneration characterized histologically
by massive accumulation of lipofuscin in the RPE cells with atrophy of the overlying
photoreceptor layer.

Parents bring their 10-month-old child to you because of chronic right-sided epiphora. They have
been massaging the right nasolacrimal sac for the past 6 months; however, the epiphora has
persisted.

Silicone stent intubation is indicated in this patient when:

dacryocystography (DCG) shows obstruction at the level of the nasolacrimal duct

nasolacrimal system probing has proven unsuccessful

massage therapy has proven unsuccessful

the patient is older than 12 months

Your answer was CORRECT


Explanation
80% to 90% of cases of congenital nasolacrimal duct obstruction resolve spontaneously by the
first year of life. Thereafter, spontaneous resolution decreases significantly. The next appropriate
step would be nasolacrimal system probing under general anesthesia. If the probing is
unsuccessful, consideration may be given to silicone stent intubation of the nasolacrimal system.
Only after multiple failed probings and silicone stent intubations would a DCR be considered for a
child.

Which is not a typical feature of Goldenhar's syndrome:


aural fistula
autosomal dominant inheritance
upper eyelid colobomas
iris colobomas
preauricular skin tags

Your answer was CORRECT


Explanation

Figure: Goldenhar syndrome

Goldenhar syndrome is characterised by:


 sporadic or AD inheritance
 pre-auricular appendages
 aural fistulas
 maxillary or mandibular hypoplasia
 hemifacial microsomia
 vertebral deformities
 lid coloboma
 Duane's syndrome
 limbal dermoids
 sensorineural deafness

A 10-year-old boy is noted to be accident prone in the dark. He complains of difficulty with vision
in bright lights. The first year ophthalmology resident mentions a peculiar pattern of pigmentation
in the retinal periphery.

What is the most likely diagnosis?

rod monochromatism

enhanced S-cone syndrome

Sjogren-Larsson syndrome

Nougaret congenital nyctalopia

Your answer was CORRECT


Explanation
Figure: Fundus photograph in enhanced S-cone syndrome.

The findings are consistent with enhanced S-cone syndrome. This is an autosomal recessive
condition with hyperfunction of the S-cones (short wavelength or blue cones) and variable
impairment of L- and M-cone functions and rod dysfunction. Patients present with nyctalopia in
childhood and hemeralopia (inability to see clearly in bright light). Some patients have
hyperpigmentation at the mid-peripheral retina.

Which children's acuity test uses crowded letters?

Keeler

Kay

Sonksen-Silver

Sheridan-Gardiner
Cardiff

Your answer was CORRECT


Explanation
The Sonksen-Silver acuity test uses crowded letters, unlike the Sheridan-Gardiner, which presents
letters singly. Kay uses matching pictures; Keeler uses a preferential grating; and Cardiff cards
have simplistic pictures in white on one end of a large grey cardboard card.

Which of the following is the MOST common presentation for retinoblastoma?


strabismus
incidental finding
orbital cellulitis
leukocoria
decreased vision

Your answer was CORRECT


Explanation
Frequency of presenting symptoms in RB:
 leukocoria: 60%
 strabismus: 22%
 decreased vision: 5%
 incidental finding: 4%
 orbital cellulitis: 2%

Which of the following statements about the epidemiology of congenital rubella infection is
TRUE?
seroconversion of a mother from negative to positive nearly guarantees transmission to the foetus

symptomatic foetal defects are uncommon, even with viraemia

maternal infection during the third trimester rarely leads to foetal infection
most pregnant women are seronegative

Your answer was CORRECT


Explanation
The later in the course of the pregnancy a mother is infected, the lower the chance of foetal
infection, and if infection occurs, the lower the likelihood of symptomatic defects. Infection early
in pregnancy leads to a high rate of abnormalities. Thanks to vaccinations, most mothers in
developed countries are seropositive.
All of the following are methods of estimating visual acuity in the preverbal child EXCEPT:
visual evoked potentials

electroretinography
preferential looking testing

optokinetic nystagmus testing

Your answer was CORRECT


Explanation
ERG is not a method to estimate visual acuity.

Acuity estimates are best with visual evoked potentials. OKN tests and preferential looking tests
are fairly close behind.

A 9-year-old presents with an acute, red, painful swelling on the lateral part of the upper lid. There
is downward and inward dystopia. The conjunctiva is inflamed at the upper lateral quadrant.

What is the most likely diagnosis?

ruptured lymphangioma

ruptured dermoid cyst

pre-septal cellulitis

acute dacryoadenitis

Your answer was CORRECT


Explanation

Figure: acute dacryoadenitis.

The findings in the question above are most in keeping with acute dacryoadenitis. A ruptured
dermoid cyst or ruptured lymphangioma do not typically cause conjunctival injection. Pre-septal
cellulitis does not cause dystopia.

Which statement is FALSE regarding juvenile idiopathic arthritis?


iridocyclitis is rarely a feature of Still's disease

the group at highest risk of developing uveitis is the pauciarticular early-onset group with
rheumatoid factor negative and antinuclear antibody negative
some sub-types of JIA are more common in males

the joints involved in patients with iridocyclitis are typically the large joints such as the knee,
ankle, and elbow

Your answer was CORRECT


Explanation
Pauciarticular, early-onset patients are most likely to develop uveitis, but they are generally ANA
positive, rheumatoid factor negative. Uveitis is rare in Still's disease, which is more common in
boys.
A 9-year-old boy presents with acute, unilateral, left-sided periocular pain, proptosis, and double
vision. Twenty-four hours later and without any treatment the pain has resolved. Periocular
ecchymosis has developed, and the double vision has stabilized. The MRI results are as shown
above.

The MOST likely diagnosis is:


capillary haemangioma
rhabdomyosarcoma
orbital abscess
lymphangioma

Your answer was CORRECT


Explanation
The MRI scan shows a layered haemorrhage in the lobular and cystic mass behind the eye. The
sinuses are clear of infection, making an orbital abscess unlikely. Acute bleeding would not be
typical of a capillary haemangioma. The haemorrhage and ecchymosis are more characteristic of
lymphangioma than rhabdomyosarcoma.

Lymphangioma usually presents in childhood and can be:


 Anterior: bluish lesion (venous blood); size increases with Valsalva
 Posterior: slowly progressive proptosis which may increase suddenly if there is a spontaneous
bleed; typically followed by ecchymosis.

The most common location for an orbital meningocele is the:

lateral canthus

lacrimal gland fossa

medial canthus
infraorbital notch

supraorbital notch

Your answer was CORRECT


Explanation
Figure: Meningo-encephalocele in an infant

A bluish bulge above the medial canthal tendon is typically a meningocoele, whereas one below is
typically a dacryocoele.

A 4-year-old girl is noted to have difficulty with visual tasks at pre-school. Ophthalmic
examination reveals translucent irides, hypopigmentation of the fundi, and foveal hypoplasia.

Each one of the following may be an associated finding EXCEPT:


recurrent pulmonary infections
oculodigital massage
bleeding diathesis
sensory nystagmus

Your answer was CORRECT


Explanation
This child has albinism, which can potentially be associated with 2 syndromes: Chediak-Higashi
and Hermansky-Pudlak.

Chediak-Higashi syndrome is characterised by:


 neutropenia causing recurrent infections
 lymphoreticular malignancies

Hermansky-Pudlak syndrome is characterised by:


 more common in Puerto Ricans
 abnormal platelets
 susceptibility to bleeding and bruising

Oculodigital massage is characteristic of Leber's congenital amaurosis

The primary site of neuroblastoma is most commonly the:


mediastinum
orbit

adrenal gland

cerebellum

Your answer was CORRECT


Explanation
Neuroblastoma most commonly arises from the adrenal gland (orbital involvement is metastatic).
Neuroblastoma can also arise from the sympathetic chain in the mediastinum.

Which of the following causes of neonatal corneal clouding has a tendency to be unilateral?

congenital hereditary stromal dystrophy

congenital hereditary endothelial dystrophy

corneal dermoids
mucopolysaccharidoses

Your answer was INCORRECT


Explanation
Corneal dermoids are typically unilateral. Other options above are bilateral.

Which is NOT a cause of heterochromic irides:

Waardenburg-Klein syndrome

Horner's syndrome

Goldenhar syndrome
Juvenile xanthogranuloma

Your answer was CORRECT


Explanation
Congenital Horner's results in a hypochromic iris on the involved side.

Waardenburg's syndrome is an autosomal dominant disorder characterised by developmental


anomalies of the eyelids, nasal root, and eyebrows along with heterochromia iridis, white
forelocks, and sensorineural deafness.

JXG is primarily a cutaneous disorder characterised by benign histiocytic proliferation. Iris


involvement is seen as richly vascularized orange nodules or as diffuse infiltration, leading to
heterochromia.

Goldenhar is not associated with heterochromia. Goldenhar is associated wth preauricular


appendages, aural fistulas, maxillary or mandibular hypoplasia, hemifacial microsomia, vertebral
deformities, notching of the upper eyelid, Duane's syndrome and limbal dermoids.

Which of the following causes of ophthalmia neonatorum may be treated topically in an otherwise
healthy infant?
Crede's prophylaxis
Herpes simplex virus
Chlamydiae subtypes D through K
Neisseria gonorrhoeae

Your answer was CORRECT


Explanation
Crede's prophylaxis or silver nitrate, which is used to prevent infectious conjunctivitis in the
neonate can itself cause a conjunctivitis presenting within 24 hours of birth (medicamentosa).
Treatment with mild topical antibiotics, or no treatment at all, is sufficient.

The other causes of neonatal conjunctivitis listed above all require systemic treatment:
 Chlamydia = oral erthromycin
 HSV = i.v. aciclovir
 Gonnorhoeae = i.v. penicillin

A couple gives birth to a child who, at the age of 7 months, is diagnosed with bilateral
retinoblastoma. There is no previous family history of the disorder.

Which one of the following statements about this situation is FALSE?


either the mother or the father has an abnormal germ cell copy of chromosome 13
the child's life expectancy is less than normal
the chance of this child having an affected brother or sister is approximately 6%
the child most likely carries one abnormal copy of chromosome 13 in each of his cells

Your answer was CORRECT


Explanation
With no family history of bilateral retinoblastoma and healthy parents, this child probably had a
germline mutation of chromosome 13. The copies of the child's parents are probably normal.

Genetic counseling in RB is complex, but the following generalizations serve as guidelines:


 Healthy parents with one affected child have a 6% chance of producing more affected children
 If two or more siblings are affected, this implies a chromosomal defect in one parent.
Therefore, each additional child has a 40% chance of inheriting the tumor (not 50% because
incomplete penetrance)
 A parent with bilateral retinoblastoma has a 50% chance of transmitting the affected
chromosome to his or her children. With 80% penetrance, 40% of offspring will be affected
phenotypically.

Which children's acuity test would be best at detecting reduced acuity in amblyopia?

Sonksen-Silver

Sheridan-Gardiner

Keeler

Cardiff

Your answer was CORRECT


Explanation
The Sonksen-Silver acuity test, unlike the other options, provides crowded letter optotypes.
Crowding is useful as it is a more sensitive test for amblyopia (i.e. it will detect reduced visual
acuity in an amblyopic eye more readily) compared to a single optotype.

Which condition is seen more commonly in girls than in boys?

congenital third nerve palsy

Duane's syndrome

congenital sixth nerve palsy

congenital oculomotor apraxia

Mobius syndrome

Your answer was CORRECT


Explanation
Congenital oculomotor apraxia is more commonly seen in boys.
Duane's syndrome is more common in girls.
There is no clear gender predilection in Mobius syndrome or in congenital third and sixth nerve
palsies.

The MOST appropriate management of newly diagnosed rhabdomyosarcoma is:

exenteration

combined radiotherapy and chemotherapy

chemotherapy

radiotherapy

Your answer was CORRECT


Explanation
Exenteration is reserved for treatment failures and recurrences.

Which of the following is NOT in the differential of a baby born with an opacified cornea?

Peter's anomaly

iridocorneal endothelial syndrome

congenital hereditary endothelial dystrophy


congenital glaucoma

congenital hereditary stromal dystrophy

Your answer was CORRECT


Explanation
Figure: ICE syndrome

ICE is typically unilateral and manifests later in life rather than at birth. Also, while ICE may
demonstrate abnormalities of the corneal endothelium (e.g. hammered silver appearance) it does
not usually cause frank corneal opacification.

A 3-month-old infant has left epiphora. Gentle compression of the lacrimal sac produces reflux of
mucus from the canaliculi on the left. There is also obviously increased tear meniscus on the left.

The next most suitable step would be:

incision and drainage of the lacrimal sac

probing and irrigation of the nasolacrimal system on the left

oral antibiotics

warm compresses to the left eye

reassurance with once daily antibiotic ointment and gentle medial canthal massage

Your answer was CORRECT


Explanation
Congenital nasolacrimal duct obstruction usually produces epiphora. The lumen of the
nasolacrimal duct is blocked near the lower ostium (valve of Hasner) by epithelial debris or a
mucosal membrane. The ostium will open spontaneously in over 90% of infants during the first 12
months of life. Gentle probing and irrigation of the nasolacrimal system are performed by 12
months of age, if the system does not open spontaneously. If probing is unsuccessful, silicone tube
intubation should be considered.
Nystagmus which has equal velocity in all directions and is symmetric in direction, amplitude, and
frequency is most appropriately described as:

uniplanar

jerk, disconjugate

pendular, conjugate

pendular, disconjugate

jerk, conjugate

Your answer was CORRECT


Explanation
Jerk nystagmus has clearly biphasic velocities, a fast in one direction and slow in the other.
Disconjugate nystagmus has different amplitude and/or frequency in one eye relative to the other.
Uniplanar nystagmus is, as its name implies, present in one plane only, usually horizontal.

The most appropriate management of neonatal inclusion conjunctivitis is:

topical erythromycin for the neonate, oral erythromycin for the mother

topical erythromycin and oral erythromycin syrup for the neonate, oral doxycycline for the
mother
topical erythromycin and oral erythromycin syrup

topical erythromycin ointment only

Your answer was CORRECT


Explanation
It is important to treat Chlamydia conjunctivitis not only locally with topical erythromycin
ointment but also with systemic treatment (erythromycin syrup) since Chlamydia can cause a
pneumonitis in the newborn. It is also important to treat the mother and any sexual partners with
oral antibiotics.

A 4-year-old child is referred by her paediatrician with a 1-week history of fever and bilateral
conjunctival infection. Other findings include a rash, erythema and oedema of the palms and soles,
periungual desquamation, cervical lymphadenopathy, infected pharynx and tongue, and fissuring
of the lips. Ophthalmic evaluation reveals a mild bilateral anterior uveitis.

Which one of the following studies is most appropriate?

chest radiograph
electrocardiogram

HLA-B27

two-dimensional echocardiography

Your answer was CORRECT


Explanation
The findings suggest Kawasaki disease, a systemic vasculitis of unknown aetiology affecting the
skin, mucous membranes, and heart. The most serious complication, vasculitis, involves the
coronary arteries resulting in formation of coronary artery aneurysms. Two-dimensional
echocardiography is indicated in these patients to evaluate ventricular function and to detect the
presence of pericardial fluid and coronary artery aneurysms.

At birth, the length of the average human eye is:

12 to 13 mm

20 to 21 mm

16 to 17 mm
8 to 9 mm

Your answer was CORRECT


Explanation
At birth, the axial length is 16 to 17 mm. Axial length increases most rapidly over the first 4 years
of life and more slowly thereafter until age 10 to 12 years.

Congenital toxoplasmosis causes all of the following EXCEPT:


hydrocephalus
retinochoroiditis
optic neuropathy
intracranial calcification

Your answer was CORRECT


Explanation
Toxoplasmosis does not affect the optic nerve.

Features of congenital toxoplasmosis in infancy and later in life:*


 Abnormal spinal fluid
 Anaemia
 Chorioretinitis †
 Seizures
 Deafness
 Fever
 Growth retardation
 Hepatomegaly
 Hydrocephalus †
 Intracranial calcifications †
 Jaundice
 Learning disabilities
 Lymphadenopathy
 Maculopapular rash
 Mental retardation
 Microcephaly
 Spasticity and palsies
 Splenomegaly
 Thrombocytopenia
 Microphthalmia
 Cataract

* Most neonates with congenital toxoplasmosis are asymptomatic as determined by routine


newborn examination.

† Signs in the classic triad suggesting the presence of congenital toxoplasmosis.

Findings consistent with the Goldenhar syndrome include all EXCEPT:


eyelid colobomas
aniridia
scoliosis
limbal dermoids

Your answer was CORRECT


Explanation
Goldenhar's syndrome is characterised by:
 preauricular appendages
 aural fistulas
 maxillary or mandibular hypoplasia
 hemifacial microsomia
 vertebral deformities (e.g. scoliosis)
 notching of the upper eyelid
 Duane's syndrome
 limbal dermoids
 sensorineural deafness
Which of the following is the MOST important prognostic factor for 5-year survival in a child
diagnosed with retinoblastoma:

tumour grade

age of patient

bilaterality

tumour size

tumour location

Your answer was CORRECT


Explanation
All of the above are factors that affect the prognosis of retinoblastoma. However the most
important of these is tumour location. If retinoblastoma is strictly intraocular, there is a 95% 5-
year survival, while if the tumour has extraocular extension (e.g. optic nerve involvement) the 5-
year survival is as low as 5%.

A 6-year-old child develops right peri-orbital erythema and redness over 2 days and is admitted
under the paediatricians for intravenous antibiotics for a presumed pre-septal cellulitis. 24 hours
after admission, there is a sudden increase in periorbital pain, swelling and conjunctival chemosis
with apparent limitation of eye movement.

What is the most likely cause?


allergic reaction to intravenous antbiotics

septicaemia

rhabdomyosarcoma

carverous sinus thrombosis

herpes zoster

Your answer was CORRECT


Explanation
The clinical picture described above is in keeping with a cavernous sinus thrombosis, which can
occur as an acute event secondary to underlying orbital or peri-orbital infection.

This question came in the FRCS (Glasgow) Part 2 in October 2014.

A 6-year-old girl develops gradual-onset, painless left vision loss. On examination, there is left
axial proptosis, a swollen left optic nerve and widespread left retinal haemorrhages. MRI scan
shows a fusiform enlargement of the optic nerve and a wide optic canal.

What is the MOST likely diagnosis?


neuroblastoma
embryonal sarcoma
spontaneous orbital haemorrhage
optic nerve sheath meningioma
optic nerve glioma

Your answer was CORRECT


Explanation

Figure: Axial T2 MRI of left ON glioma.

In the question above, the finding of fusiform enlargement is characteristic of optic nerve glioma
(as shown in the figure). By contrast, optic nerve sheath meningioma shows up as tram-track sign
on CT imaging.

Optic nerve glioma is characterised by:


 more frequent in children (90% in the first 2 decades)
 common association with NF-1 (14%-60% of all gliomas)
 can present with: vision loss, strabismus, proptosis
 optociliary shunts may be present
 can cause: ON compression and CRVO
 malignant gliomas are rare (more likely in adults than in children)
 CT/MRI: optic nerve is enlarged, and the mass may either be fusiform or exophytic in
appearance; additionally the nerve may be elongated with kinking or buckling; the lesion is iso
to hypointense on T1 MRI but hyperintense on T2
 Prognosis: many have good vision throughout the lifetime of the patient
 Treatment:
o none unless there is progressive vision loss or proptosis
o chemotherapy is usually given for children
o older children and adults usually undergo radiotherapy
o surgical removal is not possible without removing the optic nerve, and is therefore
reserved for severe proptosis or extension of tumour towards the optic chiasm
An infant with severe developmental delay has synophrys, brow hypertrichosis, low hairline,
microcephaly, ptosis and hypertelorism.

What is the most likely diagnosis?

Aicardi syndrome

Cornelia de Lange syndrome


Hallerman Streiff syndrome

Allagille syndrome

CHARGE syndrome

Your answer was CORRECT


Explanation

Figure: Cornealia de Lange syndrome

Cornealia de Lange syndrome consists of synophrys (uni-brow), brow hypertrichosis, lash


hypertrichosis, low hairline, microcephaly, long philtrum, developmental delay, among various
other developmental and musculoskeletal abnormalities. Ocular manifestations include
hypertelorism, telecanthus, ptosis, nystagmus and high myopia.
Which statement regarding cone dystrophies is FALSE?

night blindness and loss of peripheral vision are typical findings late in the course of the
disease

specific ERG findings include abnormal single-flash photopic response and flicker response

visual loss and photophobia usually precede clinically visible macular changes

cone dystrophies are characterised by decreased central vision, color blindness, and photophobia

Your answer was CORRECT


Explanation
Night blindness and loss of peripheral vision are uncommon even in advanced cases,
distinguishing this disorder from retinitis pigmentosa. Other statements above are true.

A 14-month-old infant with chronic epiphora and discharge in the left eye is brought to the
ophthalmologist by his parents. Gentle massage of the medial canthal area produces a reflux of
mucus from the left canaliculi.

The next step in management should be:


oral antibiotics

warm compresses to the left eye

incision and drainage of the lacrimal sac

reassurance with once daily antibiotic ointment and gentle medial canthal massage

probing and irrigation of the nasolacrimal system on the left

Your answer was CORRECT


Explanation
Congenital nasolacrimal duct obstruction usually produces epiphora. The lumen of the
nasolacrimal duct is blocked near the lower ostium (valve of Hasner) by epithelial debris or a
mucosal membrane. The ostium will open spontaneously in over 90% of infants during the first 12
months of life. Gentle probing and irrigation of the nasolacrimal system are performed by 12
months of age if the system does not open spontaneously as the chances of spontaneous opening at
this age are reduced. If probing is unsuccessful, silicone tube intubation should be considered.

Which of the following is frequently associated with a mild serous conjunctivitis developing
within 24 hours of birth?

Crede's prophylaxis
Neisseria gonorrhoeae

chlamydiae subtypes D through K

herpes simplex virus

Your answer was CORRECT


Explanation
The time of onset of neonatal conjunctivitis is important in determining aetiology. Silver nitrate
prophylaxis (medicamentosa) tends to present earliest (within 24 hours of birth), followed by
gonococcus (2 days), herpes simplex virus (5 to 14 days), and chlamydiae.

Important differences between neonatal inclusion conjunctivitis and adult inclusion conjunctivitis
include all of the following EXCEPT:
pseudomembranes in neonates

better response to topical therapy in neonates

more prominent follicular response in neonates


more discharge in neonates

Your answer was CORRECT


Explanation
Neonates cannot mount a follicular response because the lymphoid tissue has not fully developed.
Other statements are true.

Note: while topical treatment has a better response in neonates than adults, it is necessary to treat
both adults and neonates with inclusion conjunctivitis with systemic treatment (in neonates
because of the risk of chlamydia pneumonia). In adults the treatment is typically oral tetracycline,
doxycycline or azithromycin; in children it is typically erythromycin syrup.

A baby has been diagnosed with a rare X-linked dominant condition. There is agenesis of the
corpus callosum and multiple areas of depigmentation surrounding the optic disc.

What is the most likely diagnosis?

morning glory anomaly

MIDAS syndrome

septo-optic dysplasia

Allagille syndrome

Aicardi syndrome
Your answer was CORRECT
Explanation
Aicardi syndrome is an X-linked dominant condition. It is lethal in utero for males. There is a
characteristic fundal appearance of multiple depigmented chorioretinal lacunae surrounding the
optic disc. Other systemic abnormalities include: vertebral malformations and severe mental
retardation.

Which is TRUE regarding galactosaemia?

cataracts are inevitably progressive

disease effects are limited to the eye

it can lead to an accumulation of galactose in the lens, forming a snowflake cataract

it can result from a defect in galactokinase or galactose-1-P uridyl transferase

Your answer was CORRECT


Explanation
Galacosaemia can be Type 1 or Type 2.

Type 1 (classic) galactosaemia is autosomal recessive and is caused by deficiency of galactose-1-


phosphate uridyl transferase. Children are generally unwell (failure to thrive, hepatosplenomegaly,
CNS disease, renal disease) and the cataract is typically a central oil-droplet.

Type 2 galactosaemia is autosomal recessive and is caused by deficiency of galactokinase.


Children are generally healthy, with less systemic features and the cataract is lamellar.

Early cataract formation can be reversed with dietary intervention.

A 9-year-old has multiple blisters of the hands, feet and face only. The onset was abrupt with no
triggers found. Ocular examination reveals bilateral symblepharon, a corneal epithelial defect and
corneal vascularisation.

What is the most likely diagnosis?

mucus membrane pemphigoid

toxic epidermal necrolysis

Stevens-Johnson syndrome

epidermolysis bullosa

linear IgA disease


Your answer was INCORRECT
Explanation

Figure: bullous skin lesions in linear IgA disease

Linear IgA disease is the most common immunobullous disease in children. It is idiopathic and
skin biopsy is diagnostic. Cicatricial changes involving the conjunctiva can cause dry eye,
symblepheron, corneal vascularisation and scarring.

In children, the distribution described in this question is typical for linear IgA disease: affecting
hands, feet, face and perineum. In adults, the trunk and limbs are the most commonly affected in
linear IgA.

Stevens-Johnson syndrome (SJS) is a reasonable option for this question but the distribution of
bullous lesions leans in favour of linear IgA. Also the lack of a precipitant in this case is more in
keeping with linear IgA (which is most commonly idiopathic), whereas SJS more commonly has
an identifiable underlying aetiology (usually drug-induced or infective).

Which of the following are classified as craniofacial cleft syndromes:

1. Treacher Collins' syndrome


2. Crouzon's syndrome
3. Goldenhar's syndrome
1 and 3
1 only
1, 2, and 3
1 and 2
2 and 3

Your answer was CORRECT


Explanation

Figure: Treacher Collins syndrome

Apert's, Crouzon's and Pfeiffer's syndromes are classified as craniosynostoses. Treacher Collins'
syndrome and Goldenhar's are classified as craniofacial cleft syndromes.

Treacher Collins is characterised by:


 autosomal dominant
 malar and mandibular hypoplasia
 microstomia
 lid colobomas
 ear abnormalities
 downward displacement of the lateral canthus (antimongoloid slant)

Goldenhar's syndrome is characterised by:


 AD or sporadic
 preauricular appendages
 aural fistulas
 maxillary or mandibular hypoplasia
 hemifacial microsomia
 vertebral deformities (e.g. scoliosis)
 notching of the upper eyelid
 Duane's syndrome
 limbal dermoids
 sensorineural deafness
 other: microphthalmia, anophthalmia, ON hypoplasia, ocular colobomas

A 5 year old boy is referred by his optician with difficulty reading at school. On slit lamp
examination, you note microcysts confined to the fovea.
What is the most likely diagnosis?

rod monochromatism

congenital stationary night blindness

X-linked retinoschisis
blue cone monochromatism

Your answer was CORRECT


Explanation
X-linked retinoschisis usually presents between 3-5 years of age with reduced visual acuity. There
is bilateral maculopathy with foveal schisis, which is characterized by spoke-like radial striae.
Peripheral schisis is present in 50% and is usually infero-temporal. Schisis involves a split at the
level of the nerve fibre layer. The ERG is normal if there is maculopathy only, but is inverted if
there is peripheral schisis.

This question appeared in the 2014 FRCOphth Part 2.

Which of the following statements about congenital toxoplasmosis is FALSE:


seroconversion of a mother during pregnancy is rarely associated with placental transfer of
the organism

most pregnant women are seronegative

when placental transfer occurs, many infants remain asymptomatic

fetal infection earlier in gestation generally results in more severe involvement

Your answer was CORRECT


Explanation
Placental transfer of toxoplasma is common in the newly infected mother; however, many infected
infants will not have systemic manifestations. 70% of pregnant women are seronegative.

Most capillary haemangiomas reach their peak size at approximately what age?
6 to 12 months
12 to 24 months
5 to 7 years
2 to 3 months
3 to 5 years

Your answer was CORRECT


Explanation

Figure: Eyelid capillary haemangioma

Capillary haemangiomas generally spontaneously regress, starting in the second year of life. 40%
of tumours completely regress by age 4. 80% completely regress by age 8.

Characteristics of capillary haemangioma:


 appear a short time after birth
 composed of blood vessels that have an abnormal growth proliferation
 can be superficial or deep in the orbit
 higher blood flow compared to cavernous haemangioma (but no bruits)
 blanch with pressure (unlike port-wine nevus flammeus)
 majority: spontaneous involution
 70% will resolve before 7 years of age
 treatment: observation, topical or systemic beta-blockers (current first-line), intra-lesional
steroids, systemic steroids, sub-cutaneous interferon

Which is FALSE regarding homocystinuria?


ectopia lentis occurs in over 50%
blonde or red hair is a known association
patients are abnormal from birth, with seizures and mental retardation
patients may have osteoporosis and progressive renal dysfunction
patients may benefit from a diet low in methionine and high in cysteine

Your answer was CORRECT


Explanation
Patient are normal at birth and develop seizures, mental retardation and osteoporosis.

Homocystinuria is characterised by:


 AR inheritance
 inborn error of methionine metabolism
 elevated serum methionine and homocystine
 tall stature
 fair hair: blond or red
 osteoporosis
 scoliosis
 chest deformities
 mental retardation (50%)
 increased risk of thrombotic vascular occlusions
 lens dislocation (up to 90%), generally inferior and bilateral
 angle closure glaucoma if lens dislocates anteriorly
 treatment: diet low in methionine and high in cysteine

Which is FALSE regarding homocystinuria?


ectopia lentis occurs in over 50%
blonde or red hair is a known association
patients are abnormal from birth, with seizures and mental retardation
patients may have osteoporosis and progressive renal dysfunction
patients may benefit from a diet low in methionine and high in cysteine

Your answer was CORRECT


Explanation
Patient are normal at birth and develop seizures, mental retardation and osteoporosis.

Homocystinuria is characterised by:


 AR inheritance
 inborn error of methionine metabolism
 elevated serum methionine and homocystine
 tall stature
 fair hair: blond or red
 osteoporosis
 scoliosis
 chest deformities
 mental retardation (50%)
 increased risk of thrombotic vascular occlusions
 lens dislocation (up to 90%), generally inferior and bilateral
 angle closure glaucoma if lens dislocates anteriorly
 treatment: diet low in methionine and high in cysteine

The corneal diameter of the average healthy human infant is:


14 mm

12 mm

13 mm

10 mm

Your answer was CORRECT


Explanation
The average infant corneal diameter is 10 mm (9.5 to 10.5 mm). A corneal diameter over 12 mm
in an infant or 13 mm in a child over 2 years is termed megalocornea.

Normal adult values for corneal diameter range between 11 mm to 12.5 mm.

Which statement regarding ocular toxoplasmosis is FALSE?

less than 10% of individuals younger than 5 years of age demonstrate antibodies against
toxoplasmosis

Toxoplasma oocysts shed by cats in their faeces may remain infective for up to 1 year

retinochoroiditis typically involves the retinal periphery and infrequently involves the
macula
maternal infection later in pregnancy results in a greater risk of infection to the fetus

Your answer was CORRECT


Explanation
Toxoplasmosis retinochoroiditis, which is usually bilateral, frequently involves the macula. Other
statements above are true. Maternal infection early in pregnancy is less likely to be transmitted to
the fetus but when it does occur it is more likely than later infections to be severe.

Which is FALSE regarding retinoblastoma?


most cases have no family history

the most frequent age at diagnosis is 18 months

unilateral involvement with no family history suggests a new germline mutation

90% of cases are diagnosed by the age of 3 years

familial cases are autosomal dominant with 80% penetrance

Your answer was CORRECT


Explanation
Bilateral retinoblastoma (not unilateral) probably represents a genetic mutation, either inherited
with a family history or a new mutation if no family history. Unilateral retinoblastoma is more
likely to suggest a sporadic occurrence (2 random hits).

Which of the following patients with juvenile idiopathic arthritis (JIA) is at highest risk of
iridocyclitis?
polyarticular, rheumatoid factor positive
polyarticular, rheumatoid factor negative
pauciarticular, ANA negative
pauciarticular ANA positive

Your answer was CORRECT


Explanation
Risk factors for ocular involvement include:
 ANA positive
 rheumatoid factor negative
 pauciarticular type (90% of cases)
 early onset
 female
 involvement of lower extremity joints
 lack of involvement of the wrists

All of the following are common aetiologic agents for conjunctivitis in children EXCEPT:

Streptococcus pneumoniae

Haemophilus influenzae

Streptococcus pyogenes
Staphylococcus aureus

Your answer was CORRECT


Explanation
Streptococcus pyogenes is an infrequent cause of childhood conjunctivitis. Staplylococcus aureus,
Streptococcus pneumoniae, and Haemophilus influenzae are far more common.

Which is FALSE regarding retinopathy of prematurity?


the majority of active ROP patients develop some cicatricial complications
it is not present at birth

spontaneous regression can occur after partial retinal detachment

it usually affects the temporal periphery first

Your answer was CORRECT


Explanation
Only 20% of infants with active ROP develop cicatricial complications. The remaining regress
spontaneously.

In megalocornea which is TRUE:


it can be diagnosed in an infant if the corneal diameter exceeds 11mm

intra-ocular pressure is often elevated

the inheritance is typically autosomal recessive

the cornea is structurally normal except for its excessive diameter

Your answer was CORRECT


Explanation
In megalocornea, the corneal diameter is increased (exceeding 12mm in infants or 13 mm in
children older than 2 years) with no other detectable abnormalities. The IOP is normal, which
distinguishes the condition from buphthalmos. Megalocornea may be X-linked and is usually
bilateral.

Which is FALSE regarding childhood orbital cellulitis?

CT scan is generally preferable to MRI

it is more common in children over 5 years of age rather than under 5 years

the majority of cases emanate from maxillary sinusitis


intravenous antibiotics are the mainstay of treatment

Your answer was CORRECT


Explanation
90% of childhood orbital cellulitis cases occur secondary to sinusitis, especially of the ethmoid.
The condition is more common over the age of 5 years, with an average age of 7 years.
This question came in the 2014 FRCOphth.

A newborn boy has relatively opaque peripheral corneas, very short axial lengths and evidence of
dermal aplasia.

This consternation of signs is most typical of:


MIDAS syndrome
sclerocornea
Allagille syndrome
microcornea

Your answer was CORRECT


Explanation
MIDAS syndrome is an X-linked dominant disease characterized by:
 microphthalmia
 dermal aplasia
 sclerocornea

The parents of a child with unilateral retinoblastoma would like to know the probability of having
another child with retinoblastoma. Both parents are healthy.

What would you advise?

1%

10%

25%

50%

Your answer was CORRECT


Explanation
If a patient has unilateral retinoblastoma and the parents are healthy, the risk of the parents having
another child with retinoblastoma is 1%. The risk of the patient having a child with retinoblastoma
is 8%.

If one of the patient's parents is also affected with retinoblastoma, the risk of the parents having
another child with retinoblastoma is 40% and the risk of the patient having a child with
retinoblastoma is also 40%.

Which of the following statements about orbital lymphangiomas is FALSE?


superficial lesions may have a bluish or violaceous hue

classic presenting symptoms include spontaneous ecchymosis, and proptosis which can increase
with Valsalva

surgical intervention is indicated early in order to remove the lesion when it is small
they are primarily a disorder presenting in the paediatric age range

Your answer was CORRECT


Explanation

Surgery is not prudent unless absolutely necessary because these tumours are difficult to remove
completely and have a propensity to haemorrhage.
Reference: Paediatric Ophthalmology and Strabismus, edited by Kenneth Wright and Peter
Speigel. Springer Science (2013).

Which one of the following statements about capillary haemangiomas of the orbit and peri-orbit is
FALSE?
indications for treatment include occlusion amblyopia and significant astigmatism

characteristically blanch with pressure

more common in girls

systemic interferon may lead to involution

phlebolith formation is common

Your answer was INCORRECT


Explanation

Blood stagnation and phleboliths are characteristic of cavernous, not capillary, haemangiomas.

An 18-month-old boy presents with nystagmus and reduced visual acuity, measuring logMAR 1.0
on Cardiff cards. He is photophobic and has difficulty navigating in daylight. He is hyperopic.
What is the most likely diagnosis?
blue cone monochromatism

rod monochromatism

enhanced S-cone syndrome


deuteranopia

Your answer was CORRECT


Explanation
Achromatopsia or rod monochromatism is an autosomal recessive disorder characterized by
complete absence of cones. Patients present in the first decade with nystagmus, reduced visual
acuity, hemeralopia (inability to see clearly in bright light) and photophobia. Patients are often
hyperopic. Of the options provided, this is the most likely cause for the case described.

Blue cone monochromatism is a milder form of achromatopsia, in that there is some blue cone
function. Presenting features are similar to complete achromatopsia (rod monochromatism) except
visual acuity may be slightly better, sometimes up to 6/24. The inheritence is X-linked and
patients are usually myopic, as opposed to hyperopic in rod monochromatism.

Enhanced S-cone syndrome (Goldman-Favre syndrome) is characterised by overabundance of


blue cones, a reduced number of red and green cones, and few, if any, functional rods. Patients
experience an increased sensitivity to blue light, varying degrees of red and green colour
deficiency, night blindness occurring from early life, vision loss, and retinal degeneration.

Deuteranopia is the absence of green-sensitive cones. This results in colour vision deficiency but
has no effect on visual acuity.

This question appeared in the 2014 FRCOphth Part 2.

A 2-year-old child presents with unilateral periorbital inflammation. Which one of the following is
the LEAST likely cause?
orbital cellulitis
leukaemia
toxoplasmosis
rhabdomyosarcoma
dermoid cyst

Your answer was CORRECT


Explanation
Toxoplasmis causes intra-ocular involvement only.

Differential of paediatric periorbital inflammation:


 ruptured dermoid cyst
 rhabdomyosarcoma
 orbital cellulitis
 pre-septal cellulitis
 pseudo-tumour
 leukaemia
 eosinophilic granuloma
 infantile cortical hyperostosis

Which statement regarding epiphora in a 6-month-old is FALSE:

there is bilateral involvement in the majority of cases


the most common cause is membranous obstruction of the distal end of the nasolacrimal duct

it may be associated with hypertelorism

usually it resolves spontaneously by 1 year of age

Your answer was CORRECT


Explanation
Less than 10% of cases of congenital nasolacrimal duct (NLD) obstruction occur bilaterally.
Usually the problem resolves spontaneously by 1 year of age. The most common cause is
membranous obstruction of the distal end of the nasolacrimal duct. NLD obstruction is more
common in craniofacial abnormalities.

Which one of the following is most likely to be present on eye examination of a newborn with
congenital syphilis?

interstitial keratitis

anterior uveitis

scleritis

segmental pigmentation of the retinal periphery and chorioretinitis

Your answer was CORRECT


Explanation
Some newborns with congenital syphilis manifest active chorioretinitis; however in most, the only
evidence of chorioretinitis is segmental pigmentation of the retinal periphery with a salt-and-
pepper appearance to the fundus.

Interstitial keratitis represents an inflammatory response to treponemal antigens and usually


presents between 7 and 17 years of age.

Anterior uveitis and glaucoma may develop, but these are less common manifestations and are
usually not found in the newborn.

Hutchinson's triad for syphilis includes interstitial keratitis; widely spaced, peg-shaped teeth; and
deafness. Other systemic manifestations include saddle nose, saber shins, and rhagades (linear
scars often found around the mouth).
Which one of the following statements concerning dyslexia is NOT valid?

children with dyslexia have the same incidence of ocular abnormalities as children without
dyslexia

dyslexia is uncommon in countries such as Japan, which use one sound for each symbol

visual training, including muscle exercises, ocular pursuit, or tracking exercises, has been
proven to improve academic abilities of dyslexic children
boys are three times more likely to develop dyslexia than girls

Your answer was CORRECT


Explanation
Subnormal reading and learning may be a reason for an ophthalmologic evaluation. There is little
evidence that reading abilities result from problems in the visual system; instead, the problem lies
within the CNS and the interpretation of visual symbols. Eye defects do not cause reversal of
letters, words, or numbers. Visual training has not been proven to be helpful in dyslexia. It is
interesting to note that dyslexia is more common in languages in which certain letters or symbols
have more than one sound. Boys are more likely to have dyslexia than girls.

A 6-year-old boy is short for age, has intermittent esotropia, a beak-shaped nose, micrognathia,
and hypotrichosis. There is premature dentine eruption and enamel hypoplasia. There is bilateral
microphthalmos and membranous white cataract.

What is the most likely diagnosis?


Cornelia de Lange syndrome
Hallerman-Streiff syndrome
Allagille syndrome
Aicardi syndrome
De Morsier syndrome

Your answer was CORRECT


Explanation
Figure: Child with Hallerman-Streiff syndrome

Regarding the question above, membranous cataract is rare and usually seen in Hallerman-Streiff
syndrome. The majority of patients have dental abnormalities and proportional dwarfism. Other
diagnostic features include hypotrichosis and atrophy of skin especially on the nose.

Hallermann-Streiff syndrome is characterised by:


 sporadic occurrence
 bird-like facies with beaked nose
 alopecia
 skin atrophy
 short stature
 mandibular hypoplasia
 dental abnormalities
 large forehead
 congenital membranous cataracts
 microphthalmia
 blue sclera
 iris atrophy

Which is FALSE regarding megalocornea?


associated with progressive corneal enlargement
horizontal corneal diameter greater than 13 mm at age over 2 years is consistent with the diagnosis
usually X-linked
is known to be associated with Down's syndrome

Your answer was CORRECT


Explanation
Megalocornea is characterised by:
 X-linked recessive inheritance
 non-progressive congenital corneal enlargement
 horizontal corneal diameter over 12mm in infant or over 13 mm after 2 years of age
 typically an isolated abnormality
 systemic associations include:
o Down's syndrome
o Marfan's syndrome
o Alport's syndromes
o craniosynostosis
o facial hemiatrophy

The most common aetiology of posterior uveitis in the paediatric population is:
sarcoidosis

toxoplasmosis

toxocariasis

syphilis

idiopathic

Your answer was CORRECT


Explanation
Up to 50% of paediatric patients with posterior uveitis have ocular toxoplasmosis.

Which is TRUE regarding retinitis pigmentosa?


the initial visual field defect is a ring scotoma
signs and symptoms typically precede ERG abnormalities
retinal pigmentary changes in the midperiphery are always present
the X-linked form is least common but most disabling

Your answer was CORRECT


Explanation
RP is characterised by:
 1:3500 incidence
 50% sporadic
 50% inherited: 22% AD, 16% AR, and 9% X-recessive
 X-linked most severe form
 ERG abnormalities precede signs and symptoms
 Early ERG: normal cone response, decreased scotopic b-wave
 Late ERG: nonrecordable ERG
 Early VF: inferotemporal scotoma
 Late VF: ring or annular scotoma
 Findings: attenuation of the retinal vessels, midperipheral pigmentary changes, and optic disc
pallor
 Associations: include myopia, vitreous opacities, posterior subcapsular cataract, retinal
pigment epithelial atrophy, cystoid macular oedema, glaucoma, and keratoconus

Which statement regarding congenital impatency of the nasolacrimal system is FALSE?


the defect in canalisation is within the intraosseous portion of the nasolacrimal duct
common symptoms include epiphora and mucus discharge

if canalisation has not spontaneously occurred by the age of 12 to 15 months, it is unlikely to do so

acute dacryocystitis is uncommon

congenital impatency may mimic a medial canthal haemangioma

Your answer was CORRECT


Explanation
Dacryocoele, or congenital mucocele of the nasolacrimal sac, typically has a bluish-red hue,
similar to a haemangioma, and it results from chronic obstructive tear stasis. NLD impatency
tends to cause recurrent conjunctivitis while dacryocystitis is uncommon. Obstruction is nearly
always at the inferior extreme of the nasolacrimal duct, at the valve of Hasner (within the inferior
nasal meatus). If it persists beyond 1 year of age, surgical intervention is generally required.

All of the following should be considered in the differential diagnosis of vitreous haemorrhage in
a 9-year-old child EXCEPT:

trauma

melanocytoma
juvenile X-linked retinoschisis

pars planitis

Your answer was CORRECT


Explanation
A melanocytoma is an elevated, deeply-pigmented lesion most often found at the optic nerve head.
Histologically, it appears as a benign proliferation of melanocytic cells. They are not known to
cause vitreous haemorrhage.

Which statement is LEAST accurate with regard to a paediatric dacryocystocoele?

definitive therapy includes nasolacrimal duct probing

nasolacrimal duct probing should be performed after 6 months of age

if the condition is infected, IV antibiotics are needed


differential diagnosis includes haemangioma, encephalocele, and dermoid cyst

Your answer was CORRECT


Explanation
A dacryocystocele presents at birth as an enlargement of the lacrimal sac due to a blockage at the
valve of Hasner. Its appearance has been confused with haemangioma, encephalocele, and
dermoid. If these cysts become infected, sepsis is possible; therefore early NLD probing in
neonates and IV antibiotics are often necessary.

Posterior uveitis in children may be due to all EXCEPT:


Posner Schlossman syndrome
Toxoplasmosis
ARN syndrome
Behcet's disease
Diffuse unilateral sub-acute neuroretinitis (DUSN)

Your answer was CORRECT


Explanation
Posner-Schlossman syndrome does not cause posterior uveitis.

Posner-Schlossman syndrome is characterised by:


 unilateral ocular pain
 mild anterior uveitis
 elevated IOP
 self-limiting episodes (may require transient course of IOP meds and topical steroids)
 glaucoma may result from recurrent attacks

A patient with pigmentary retinopathy had an extra finger removed as a child. All his fingers are
stubby. He is obese with type 1 diabetes and renal dysfunction. There is bull's eye maculopathy.

What is the diagnosis?


Refsum syndrome
Bardet-Biedl syndrome
Cockayrne syndrome
Alstrom syndrome

Your answer was CORRECT


Explanation
Figure: characteristics of Bardet-Beidl syndrome. Figures (a–d) demonstrate typical facial features
including: deep-set eyes, hypertelorism, downward slanting palpebral fissures, a flat nasal bridge,
small mouth, malar hypoplasia and retrognathia; (e) brachydactyly and scars from excision of
accessory digits; (f) dental crowding; (g) high-arched palate; (h) rod-cone dystrophy.

Bardet-Beidl syndrome is characterised by:


 AR inheritence
 pigmentary retinopathy
 obesity
 brachydactyly or polydactyly
 dental anomalies
 hypogenitalism
 renal disease
 cardiac disease
 mental handicap
An 8-year-old boy complains of blurred vision for 1 week. Fundus findings are shown in Figure A
above. What information from the history is in keeping with the most likely diagnosis?

blunt trauma to the orbit

prematurity with low birth weight

juvenile-onset diabetes

a pet cat at home

Your answer was CORRECT


Explanation
This patient has neuroretinitis, which has been linked with viral infections (mumps, influenza,
varicella) and other infections such as Cat-scratch and leptospirosis.

An infant presents for evaluation with concerns about visual development. Which one of the
following signs is LEAST worrisome?

paradoxic pupillary response to light

eye rubbing
eye-popping reflex
gazing at bright lights

Your answer was CORRECT


Explanation
The eye-popping reflex is a normal neonatal reflex which involves a pronounced widening of the
palpebral fissures after an abrupt decrease in ambient illumination primarily, or after loud noises.
It is present within the first 3 weeks of life in 75% of infants born.

Eye pressing or gouging and light gazing are abnormal behavioral mannerisms which occur in
visually-impaired children.

A paradoxical pupillary response is rare but when present, it is highly suggestive of congenital
stationary night blindness, achromatopsia, or optic nerve hypoplasia. A paradoxic response refers
to an immediate constriction during the first 20 seconds after room lights are turned off, followed
by a slow dilation after 1 minute.

A 12-month-old child is referred with delayed visual maturation. On examination, there are
roaming eye movements and nystagmus. Pupil reactions to light are diminished. The fundus
appears normal, but the ERG is markedly abnormal showing almost entirely extinguished
responses.

What is the diagnosis?

congenital stationary night blindness

achromatopsia

Usher's syndrome

Leber's congenital amaurosis

Your answer was CORRECT


Explanation
An extinguished ERG with a normal fundus is typical of early Leber's congenital amaurosis.

A child with cataract, haematuria and hearing loss is most likely to have:
Alport's syndrome
Lowe's syndrome
incontinentia pigmenti
Fabry's disease
homocystinuria
Your answer was CORRECT
Explanation

Alport's syndrome can be inherited in both X-linked recessive (80%) and X-linked dominant (5%)
forms. Where X-linked recessive, heterozygous females may still experience haematuria - a
separate condition termed 'thin basement membrane nephropathy.' Reference: NIH National
Library of Medicine.

Alport's is caused by a defect in Type 4 (basement membrane) collagen. Clinically, it causes:


 glomerular basement membrane nephritis and renal failure
 hearing loss
 anterior lenticonus (shown in the image above)
 cortical cataract
 juvenile arcus
 posterior polymorphous dystrophy
 dot-fleck retinopathy
Lowe's syndrome causes aminoaciduria.

Systemic mortality caused by Kawasaki's disease is most frequently due to:


myocardial infarction
stroke
acute renal failure
respiratory failure
acute adrenal insufficiency

Your answer was CORRECT


Explanation
The vasculitis of Kawaskai's may involve the coronary arteries and lead to coronary artery
aneurysm or occlusion.

Features of Kawasaki's disease:


 fever lasting over 5 days
 injected pharynx
 lymphadenopathy
 bilateral conjunctivitis
 mucous membrane fissures
 strawberry tongue
 desquamating rash of palms and soles
 non-specific rash
 complications: coronary artery aneurysm
 Treatment: aspirin

A child with alopecia and bird-like facies is most likely to have:


incontinentia pigmenti
Hallermann-Streiff syndrome
Fabry's disease
ichthyosis
myotonic dystrophy

Your answer was CORRECT


Explanation
Figure: Child with Hallerman-Streiff syndrome

Hallermann-Streiff syndrome is characterised by:


 bird-like facies with beaked nose
 alopecia
 short stature
 mandibular hypoplasia
 large forehead
 congenital cataracts
 microphthalmia
 blue sclera
 iris atrophy

A 12-year-old girl has bulging and ptosis of both upper eyelids and repeated episodes of eyelid
inflammation and swelling.

What is the most likely diagnosis?


blepharochalasis
blepharospasm
dermatochalasis
steatoblepharon

Your answer was CORRECT


Explanation
Figure: Blepharochalasis

Blepharochalasis is characterised by:


 a rare, inherited condition
 typically affects young females more than males
 repeated bouts of eyelid inflammation and oedema
 recurrent lid swelling eventually leads to:
o ptosis secondary to dehiscence or attenuation of the levator aponeurosis
o anterior bulging of the lacrimal gland and orbital fat

Note:
Dermatochalasis refers to the excess eyelid skin that occurs with aging. Steatoblepharon refers to
the bulging of orbital fat that also commonly occurs with aging or attenuation of the orbital
septum.

Which of the following visual acuity tests is most likely to be suitable for a normally-developing
4-year-old child?

Cardiff cards

Teller cards

Keeler preferential looking cards

Sonksen cards

Your answer was CORRECT


Explanation
The Teller and Keeler cards are preferential looking cards containing gratings. Cardiff cards are
preferential looking cards, which contain simplistic images in light-grey (e.g. a duck) at one end of
a grey card. Preferential looking cards are useful for 0 months to 2 years.

The Kay, Sheridan-Gardiner, Sonksen and Keeler logMAR tests contain pictures and/or letter
optotypes for matching or naming in single or crowded optotypes (good for 2+ years for Kay,
which has pictures, and 3+ years for other options depending on child maturity and co-operation
in naming letters).

A 9-year-old boy has a 3-week history of a rapidly progressing supero-nasal mass that does not
affect vision. Examination shows proptosis pushing the right eye down and out.

The best management includes all of the following EXCEPT:


CT scan
anterior orbitotomy with biopsy
MRI scan
observation

Your answer was CORRECT


Explanation
Rapid evolution of unilateral proptosis in a child should be considered rhabdomyosarcoma until
proven otherwise. This finding necessitates a biopsy and then radiation and chemotherapy.

Causes of supero-nasal masses include:


 rhabdomyosarcoma (needs to be excluded)
 myocele
 mucopyocele
 encephalocoele
 neurofibroma

All are TRUE of ophthalmia neonatorum EXCEPT:


Herpes simplex is an important cause in infants under 4 weeks of age
Chlamydia conjunctivitis can be effectively managed with topical antibiotics alone
staining of the corneal epithelium is a vital part of the examination
intracytoplasmic inclusion bodies on Giemsa stain will provide a definitive diagnosis

Your answer was CORRECT


Explanation
Ophthalmia neonatorum can be the manifestation of several different infections. The differential
diagnosis includes:
 Chlamydia
 Neisseria gonorrhea
 herpes simplex
 other viruses and bacteria
Chlamydia can be diagnosed with Giemsa stain, which reveals intracytoplasmic inclusion bodies.
Chlamydia conjunctivitis can be treated with topical erythromycin, but the child must also be
treated with systemic therapy to prevent an associated Chlamydia pneumonitis. The pneumonia
usually has its onset 3 to 13 weeks later.

Neisseria gonorrhea is well known for its potential to penetrate an intact corneal epithelium and
cause a corneal perforation.

Herpes simplex type 2 is an important cause of neonatal conjunctivitis with keratitis with
involvement of the corneal epithelium. Untreated neonatal herpes keratitis can cause corneal
scarring and dense amblyopia.

A 2-year-old has infantile spasm, corpus collosum agenesis, choroidal lacunae, optic disc
hypoplasia, microphthalmos and persistent pupillary membrane.

What is the diagnosis?


Hallerman Streiff syndrome
Cornelia de Lange syndrome
Aicardi syndrome
Allagille syndrome

Your answer was CORRECT


Explanation
The features suggest Aicardi syndrome which is characterised by:
 X-linked dominant condition
 fatal if inherited by males
 infantile spasm
 severe mental retardation
 corpus callosum agenesis
 optic disc hypoplasia
 multiple depigmented chorioretinal lacunae surrounding the optic disc
 microphthalmos
 persistent pupillary membrane
 vertebral malformations

Note: optic disc hypoplasia and agenesis of the septum pellucidum with malformation of corpus
collosum can occur in De Morsier's syndrome, which can be associated with hypothalamic
dysfunction (causing stunted growth)

A 7-year-old child is referred by her opticians with elevated white fibrous tissue seen emanating
from and hovering above the right optic disc.

What is the most likely diagnosis?

Mittendorf's dot

regressed retinopathy of prematurity


Bergmeister's papilla
myelinated nerve fiber

area of Martegiani

Your answer was CORRECT


Explanation
Bergmeister's papilla is an anomaly of posterior hyaloid vessel regression with associated glial
tissues emanating from the disc.

Which of the following causes of ophthalmia neonatorum may be treated with oral antibiotics?

Crede's prophylaxis

Herpes simplex virus

Neisseria gonorrhoeae

Chlamydiae subtypes D through K

Your answer was CORRECT


Explanation
Chlamydia is treated with oral antibiotics such as erythromycin. Neonatal conjunctivitis caused by
HSV and Gonnorhoeae require i.v. aciclovir and i.v. penicillin respectively.

Intermediate uveitis in children may be associated with all of the following EXCEPT:
Sarcoidosis
Lyme disease
VKH
Toxoplasmosis
Tuberculosis

Your answer was CORRECT


Explanation
VKH is not considered in the differential diagnosis of intermediate uveitis (pars planitis) because
the main site of disease in VKH is the choroidal stroma, with exudative retinal detachments,
thickening of the choroid on ultrasound and OCT imaging and chronic chorioretinal atrophy
(sunset glow fundus). An isolated vitritis does not occur in VKH.

Differential of pars planitis includes:


 MS
 Lyme
 Sarcoid
 TB
 Syphilis
 Toxoplasmosis

All of the following are typical findings in a patient with craniosynostosis EXCEPT:
A-pattern exotropia
telecanthus
mid-facial hypoplasia
proptosis

Your answer was CORRECT


Explanation
V-pattern exotropia (not A-pattern) is associated with craniosynostoss.

Craniosynostosis (premature closure of cranial sutures) is characterised by:


 midfacial hypoplasia
 V-pattern exotropia
 proptosis
 telecanthus
 oral and dental problems
 respiratory problems

A 7-year-old child presents with sudden onset painful unilateral axial proptosis with overlying lid
erythema developing over 2 weeks. She is apyrexial. A CT scan shows a non-enhancing, poorly
defined mass of homogenous tissue density.

This presentation would be most typical of which condition?

rhabdomyoscarcoma

lymphangioma

capillary haemangioma

orbital varices

Your answer was CORRECT


Explanation
Rhabdomyosarcoma is a very rare disorder but it is an important differential in childhood, sudden-
onset, painful, unilateral proptosis. The peak incidence is age 7-8 years and CT findings are as
described in the case above. In children under the age of 4, capillary haemangioma and
lymphangioma are more likely causes of proptosis but these conditions typically present over a
longer course (unless there is haemorrhage within the lesion). In children over the age of 10 years,
orbital cellulitis, dermoid cyst and non-specific orbital inflammation are more common causes of
proptosis.

This question came in the 2014 FRCOphth.

Hutchinson's triad for congenital syphilis includes:


rhagades, interstitial keratitis, and hepatosplenomegaly
pigmentary retinopathy, interstitial keratitis, and peg-shaped teeth
interstitial keratitis, cataract, and pseudoretinitis pigmentosa
pigmentary retinopathy, eighth nerve deafness, and interstitial keratitis
peg-shaped teeth, eighth nerve deafness, and interstitial keratitis

Your answer was CORRECT


Explanation
Hutchinson's triad for congenital syphilis includes:
 interstitial keratitis
 widely spaced, peg-shaped teeth
 deafness

Other systemic manifestations of congenital syphilis include saddle nose, saber shins, and
rhagades (linear scars often found around the mouth), but these do not form part of the triad.

A 6-month-old child has a capillary haemangioma of the upper lid. On examination, the upper lid
margin in the affected eye rests at the lower pupil border. What approach is most appropriate for
this child?

radiotherapy

observation

interferon-alpha-2a

oral propranolol

Your answer was CORRECT


Explanation
The treatment of capillary haemangiomas depends on many factors, including vision, location,
size, and systemic condition. Lesions typically spontaenously regress so small lesions without
visual compromise can be observed and vision can be monitored.

Larger lesions can affect the patient's coagulation status by decreasing platelets.
Thrombocytopenia secondary to sequestration of platelets in the tumor is termed the Kassabach-
Merritt syndrome. A large lesion can also compromise vision by blocking the visual axis. In either
of these cases, treatment is indicated.

Corticosteroids can be administered topically, orally, or by intralesional injection. Complications


of steroid injection include hypopigmentation and thinning of the skin and orbital fat, and very
rarely central retinal artery occlusion. Other treatment options include systemic and topical
propranolol, which has recently shown significant efficacy with less side-effects than steroids and
is leading to a change in the treatment paradigm of these lesions. Interferon-alpha-2a once held
promise although it is no longer regularly used due to potential side-effects including spastic
diplegia. Surgical excision is an additional treatment option, but is usually not first-line.

A 12-year-old boy with no significant past medical history presents to ophthalmology following a
referral from his opticians who were concerned about fundal appearances. On examination, you
find vitreous degeneration, areas of white without pressure in the retinal periphery, peripheral
retinal vascular toruosity and fibrovascular proliferation with a ridge formation temporally. You
also note temporal dragging of the disc and macula.

What is the most likely diagnosis?

Wagner syndrome

Favre-Goldmann disease

Eales disease

retinopathy of prematurity

familial exudative vitreoretinopathy

Your answer was CORRECT


Explanation

Figure: FEVR showing ischaemia in the peripheral retina on FFA with secondary fibrovascular
proliferation which is typically more marked temporally and can lead to temporal disc dragging
and retinal detachment

The description is most in keeping with FEVR, which is characterised by failure of vascularisation
of the temporal retinal periphery, similar to that seen in ROP but in the absence of low birth
weight or prematurity. Complications include tractional retinal detachment and massive sub-
retinal exudation. Prognosis is generally poor.

Which is TRUE regarding juvenile idiopathic arthritis?

uveitis is uncommon in the pauciarticular group

the uveitis causes pain and conjunctival injection

uveitis usually requires systemic steroid treatment

macular oedema is a cause of visual loss

Your answer was CORRECT


Explanation
Pauciarticular arthritis (confined to four or fewer joints) is associated with higher risk of uveitis in
patients with JIA, as is ANA positivity and female sex.

JIA uveitis usually occurs in the absence of pain or redness, with the main presenting feature
being loss of vision, which can occur due to cataract, CMO, glaucoma and band keratopathy.
Topical steroid treatment is usually sufficient.

All of the following are more likely in paediatric orbital pseudotumour compared to the adult
variety EXCEPT:

minimal periocular pain


systemic symptoms and signs of malaise, fever and vomiting

bilateral involvement

uveitis

peripheral eosinophilia

Your answer was CORRECT


Explanation
Orbital pseudotumour is generally quite painful for both children and adults. Other options are
true.

Which of the following statements regarding neuroblastoma is INCORRECT:

15% of childhood deaths are due to neuroblastoma

the average age of diagnosis of orbital neuroblastoma is 8 years old


Horner's syndrome may occur in neuroblastoma

neuroblastoma may originate in the adrenal gland, mediastinum, or neck

Your answer was CORRECT


Explanation
The average age of diagnosis of neuroblastoma is 2 years old; rhabdomyosarcoma commonly
presents at age 8.

A 10-year-old Caucasian girl presents with right eye floaters and blurred vision. On examination,
she has a mild anterior chamber inflammatory reaction and corneal changes suggestive of band
keratopathy. Dilated fundus examination reveals whitish fibrous material in the inferior periphery.

Which of the following is associated with this condition?


unilaterality in the majority of cases
HLA-B27
anterior subcapsular cataracts
multiple sclerosis

Your answer was CORRECT


Explanation
Figure: Indentation examination of the peripheral retina shows 'snowbanking' of the pars plana,
which is pathognomic of pars planitis

The presentation in the question above is suggestive of pars planitis. Multiple sclerosis has been
associated with this condition in approximately 5% of cases, and the condition is bilateral
approximately 80% of the time.

Ocular features of pars planitis include:


 usually bilateral
 mild AC reaction
 vitritis
 snowballs
 snowbanking
 posterior subcapsular cataracts
 macular epiretinal membranes
 retinal phlebitis
 retinal neovascularisation (rare)
 vitreous haemorrhage (rare)
 tractional or rhegmatogenous RD
 subtle disc oedema
 cystoid macular oedema
 glaucoma
 associations: multiple sclerosis, sarcoid, Lyme

The probability of a patient who survives bilateral retinoblastoma giving birth to a child with
retinoblastoma is:
6%

25%

1%

80%

40%

Your answer was CORRECT


Explanation
A parent with bilateral retinoblastoma has a 50% chance of transmitting the affected chromosome
to his or her children. With 80% penetrance, 40% of offspring will be affected phenotypically.

Which is the most appropriate visual acuity test for a healthy 3-month-old baby:

forced preferential looking


Sheridan-Gardner chart

Kay pictures

Cardiff acuity cards

Your answer was CORRECT


Explanation
Less than 1 year, forced preferential looking cards such as Keeler are most appropriate.

Between 1-year-old to 2-year-old Cardiff acuity cards are suitable.

Kay pictures are used for children with sufficient language skills to name the pictures (usually age
2).

Sheridan-Gardner or alternatives such as the Sonksen (cards each containing a single letter or
crowded letters) are useful for children age 3 or older.

A child contracts Toxocara chorioretinitis. Which one of the following statements regarding this
condition is FALSE?
the infection often manifests as an eosinophilic granuloma
the infection can present as an apparent exotropia in which there are no refixation movements on
alternate cover testing
calcification is frequently present on CT imaging
the infection may present with either anterior or posterior segment involvement

Your answer was CORRECT


Explanation
Features of Toxocara canis:
 nematode larva
 usual host is the dog or cat
 ingested by children
 eosinophilic granuloma at posterior pole
 slit-lamp appearance can be mistaken for retinoblastoma
 unlike RB, Toxocara does NOT have calcification on CT
 temporal disc dragging

All of the following are true of malignant hyperthermia (MH) EXCEPT:


elevated body temperature is a late sign of MH
MH can be triggered by succinylcholine, enflurane, and lidocaine
MH may be manifested by trismus and tachycardia during induction
mortality rate from MH is less than 20%

Your answer was CORRECT


Explanation
MH is triggered by succinylcholine, halothane, enflurane, and isoflurane. It is not triggered by
local anesthetics.

Malignant hyperthermia is characterised by:


 incidence of 1:6000 and 1:30000
 more common in children
 triggered by succinylcholine, halothane, enflurane, and isoflurane
 mortality rate now less than 10%
 creatine phosphokinase is elevated in up to two-thirds of patients
 the earliest sign is tachycardia or elevated end-tidal carbon dioxide
 other early signs:
o unstable blood pressure
o tachypnea
o sweating
o muscle rigidity
o cyanosis
o dark urine
 a rise in temperature is a late sign

Which of the following is typically associated with a hyperacute purulent conjunctivitis


developing 2 days after birth?

Neisseria gonorrhoeae
herpes simplex virus

chlamydiae subtypes D through K

Crede's prophylaxis

Your answer was CORRECT


Explanation
The time of onset of neonatal conjunctivitis is important in determining aetiology. Silver nitrate
prophylaxis (medicamentosa) tends to present earliest (within 24 hours of birth), followed by
gonococcus (2 days), herpes simplex virus (5 to 14 days), and chlamydiae.

An 8-year-old boy presents for evaluation of decreased vision. Ophthalmologic examination


reveals degenerative changes involving the vitreous and retina and an optically empty vitreous.

Which one of the following is the LEAST likely diagnosis?


Wagner dystrophy
Stickler syndrome
Goldmann-Favre dystrophy
Kearns-Sayre syndrome

Your answer was CORRECT


Explanation
Kearns-Sayre is not associated with an optically empty vitreous.

Kearns-Sayre syndrome is characterised by:


 mitochondrial inheritence
 ragged red fibres on muscle biopsy
 presentation in 1st or 2nd decades
 ptosis
 external ophthalmoplegia
 cardiac conduction defects
 deafness
 diabetes
 short stature
 pigmentary retinopathy

Crouzon's syndrome is associated with all of the following ophthalmic findings EXCEPT:
nystagmus
telecanthus but not hypertelorism
exotropia
optic atrophy

Your answer was CORRECT


Explanation

Figure: child with Crouzon's syndrome.

Crouzon's is associated with hypertelorism (widely separated eyes), not just telecanthus (in which
the medial canthi are widely separated but the eyes themselves have a normal inter-pupillary
distance).

Crouzon's syndrome is characterised by:


 craniosynostosis: premature fusion of cranial and saggital sutures
 AD inheritance
 short antero-posterior head distance and wide cranium
 mid-facial hypoplasia
 parrot beak nose
 frog-like facies
 proptosis from shallow orbits
 hypertelorism
 exposure keratopathy
 optic atrophy
 V-pattern exotropia
 low-frequency features: cataract, nystagmus, microcornea, megalocornea, keratoconus, ectopia
lentis, blue sclera and glaucoma

What is the pattern of inheritance of Best diseases?

autosomal dominant

mitochondrial

autosomal recessive

X-linked recessive

Your answer was CORRECT


Explanation
Best disease is an autosomal dominant hereditary macular degeneration.

A 4-month-old baby boy is referred by his paediatrician and health visitor who are concerned
about the child's visual development. This is the parent's first child. The child is otherwise healthy
and appears to be growing and developing normally for age. On examination, there are roving eye
movements with no fixing or following. Fundoscopy reveals mid-peripheral pigmentary changes
and a Bull's eye-type maculopathy at the fovea. An ERG reveals extinguished responses.

What is the most likely diagnosis?


Leber's congenital amaurosis
Best macular dystrophy
Juvenile neuronal ceroid lipofuscinosis (Juvenile Batten's disease)
Leber's hereditary optic neuropathy

Your answer was CORRECT


Explanation
Leber's congenital amaurosis is characterised by:
 very poor visual prognosis
 extinguished ERG
 oculodigital syndrome: eye rubbing causes enophthalmos from resorption of orbital fat
 fundi may be initially normal
 fundal findings may include:
o chorioretinal atrophy
o pigmentary retinopathy
o macular coloboma-like atrophy
o Bull's eye-type maculopathy

Juvenile Batten's disease (ceroid lipofuscinosis) can also present with a Bull's eye maculopathy,
but it presents later (age 2 to 10) and there are associated neuro-behavioural changes. Best's
disease is another condition that may manifest as a Bull's eye-type maculopathy but presentation
occurs in later childhood or adolescence and pigmentary retinopathy is not a feature.

This question came in the FRCS (Glasgow) Part 2 in October 2014.

A 4-year-old child is under paediatricians with aminoaciduria. There is developmental delay,


hypotonia, abnormal skull shape and lamellar cataract bilaterally.

What is the most likely diagnosis?


Alport syndrome
Allagille syndrome
Aicardi syndrome
Lowe syndrome
Fabry disease

Your answer was INCORRECT


Explanation
Lowe syndrome is associated with:
 X linked recessive inheritance
 mental handicap
 Fanconi syndrome of proximal renal tubules
 muscular hypotonia
 congenital cataract
 microphakia
 posterior lenticonus
The picture shown is most likely caused by:

congenital cataract

normal optic nerve

retinoblastoma

Coat’s disease

Your answer was CORRECT


Explanation
The most common cause of apparent leucocoria on flash photography is a normal optic nerve.
However, the differential of leucocoria must be considered and excluded.

This has been well described in the literature: e.g. Am J Ophthalmol. 2003 May;135(5):709-11.

This question came in the 2014 FRCOphth.

The association of optic disc hypoplasia with short stature and midline developmental anomalies
is most in keeping with:

MIDAS syndrome

morning glory anomaly

Aicardi syndrome
Allagille syndrome

de Morsier syndrome

Your answer was CORRECT


Explanation
de Morsier's syndrome (septo-optic dysplasia) is the association of optic nerve hypoplasia,
agenesis of the corpus callosum and absence of the septum pellucidum. There is also a common
association with pituitary deficiency with growth hormone deficiency being the most common.

All are characteristics of patients with Lowe's syndrome EXCEPT:


bilateral congenital cataracts
X-linked recessive inheritance
glaucoma
glomerulonephritis

Your answer was CORRECT


Explanation
Lowe's is associated with renal tubular acidosis due to Fanconi syndrome, not glomerulonephritis.

Lowe syndrome is associated with:


 X-linked recessive inheritance
 mental handicap
 Fanconi syndrome of proximal renal tubules
 aminoaciduria
 muscular hypotonia
 congenital cataract
 microphakia
 posterior lenticonus
 glaucoma in 50%

The typical colour of a juvenile xanthogranuloma is:


blue

orange

pearly

black

speckled grey

Your answer was CORRECT


Explanation

The combination of high lipid content (yellow) and vascularity (red) leads to the colour of these
benign growths.

Which children's acuity test presents single letter optotypes?

Cardiff

Keeler

Sonksen-Silver

Sheridan-Gardiner

Your answer was CORRECT


Explanation
The Sheridan-Gardiner acuity test uses single letters, unlike the Sonksen-Silver test, which
presents crowded letters. Kay uses matching pictures (either single or crowded); Keeler uses a
preferential grating; and Cardiff cards have simplistic pictures in white on one end of a large grey
cardboard card.

All cases of Peters' anomaly share which feature:


ectopia lentis

polycoria

central absence of Descemet's membrane

cataract
Your answer was CORRECT
Explanation

Figure: Peter's anomaly

All cases of Peter's anomaly share central absence of Descemet's membrane and endothelium with
corneal leukoma. An anterior cataract or dislocated lens may be present but is not necessary for
the diagnosis.

Peter's anomaly is typically sporadic and it is bilateral in two-thirds of cases. It is characterised by


absence of posterior corneal tissue and leukoma, ±iris adhesions to leukoma, ±lens-corneal
adhesions. Opacification of the cornea leads to an amblyogenic effect on a developing infant.
Peters' plus syndrome includes short disproportionate stature, developmental delay, dysmorphic
facial features, cardiac, genito-urinary, and central nervous system malformation. These systemic
findings are seen in up to 60% of patients. Peters’ is also associated with many other ocular
pathologies including glaucoma, sclerocornea, corectopia, iris hypoplasia, cataract, ICE syndrome,
aniridia, iris coloboma, persistent fetal vasculature and microcornea.

Following unsuccessful probing of a congenitally impatent nasolacrimal system in a 13-month-old


girl, the next therapeutic step would be:

warm compresses

dacryocystorhinostomy

silicon tube intubation


antibiotic ointment and medial canthal massage

Your answer was CORRECT


Explanation
If probing of a congenitally impatent nasolacrimal system is unsuccessful, silicone tube intubation
should be considered and is effective in 85% of the cases. DCR should be delayed until 3 years of
age for long-standing success.

Which statement regarding ocular adnexal dermoid cysts is FALSE?


generally they do not enlarge after the first year of life

radiography of orbital lesions generally demonstrates bony excavation

the most common location is the superonasal orbital rim

rupture may lead to an orbital cellulitis-like picture

they are formed by the entrapment of ectodermal tissue during embryologic development

Your answer was CORRECT


Explanation

The most common location of an adnexal dermoid cyst is the superotemporal orbital rim, in
association with the zygomaticofrontal suture line, as shown above.

Special considerations when performing penetrating keratoplasty in children include all of the
following EXCEPT:

oversized grafts

early removal of sutures post-operatively

scleral support during surgery

frequent post-operative follow-up

Your answer was CORRECT


Explanation
PK in children is very challenging. The eye wall is extremely flaccid, and scleral support is
crucial. Anterior bulging of the lens-iris diaphragm is common, and many surgeons use a small-
sized graft to reduce this problem as well as to reduce the chance of peripheral synechiae
formation. Frequent post-operative follow-up and early suture removal (as early as 2 to 4 weeks
post-operatively in neonates) help reduce the incidence of neovascularization of the graft.

What is the most common site of retinoblastoma spread outside the eye?

lungs

liver

CNS
lymph nodes

bone

Your answer was CORRECT


Explanation
Direct spread into the optic nerve and CNS is most common. Bone metastasis is seen in over half
of metastatic cases. Lymph node metastases are the second most common type.

A 10-year-old girl presents with acute, unilateral, left-sided periocular pain, proptosis, and double
vision.

Which condition should NOT be considered in the differential diagnosis?

sinusitis with orbital abscess

cavernous haemangioma

traumatic retrobulbar hemorrhage

orbital lymphangioma

Your answer was CORRECT


Explanation
A cavernous haemangioma is a slow-growing lesion that creates proptosis over years or months.
Note: orbital lymphangioma may produce acute proptosis when there is a haemorrhage from
interstitial capillaries. Acute infection and retrobulbar haemorrhage (e.g. following trauma) can
both present with rapidly developing proptosis.
A 10-year-old girl with a background history of hearing impairment presents to the
ophthalmologist with difficulty reading small print at school and nyctalopia. On examination,
there are areas of RPE hyperpigmentation scattered throughout the mid-peripheral fundus.

Which of the following investigations is most appropriate in this case?

VEP

dark adapted ERG


EOG

ICG

Your answer was CORRECT


Explanation
Pigmentary retinopathy suggests the possibility of rod-cone dystrophy, which needs to be
excluded by electrophysiology. Dark-adapted ERG is the most specific test for rod function.

This question came in the FRCS (Glasgow) October 2014 exam.

Which is TRUE of orbital lymphangioma:


the most common primary orbital malignancy in children
the most common cause of proptosis in children
an orbital vascular lesion that will involute after intralesional corticosteroids
a tumor that may enlarge with upper respiratory infections

Your answer was CORRECT


Explanation

Figure: Orbital lymphangioma

Lymphangiomas may increase in size during viral infection, presumably caused by the
lymphocytic components of the tumor. Orbital cellulitis is the most common cause of proptosis in
children; lymphangioma is not a malignant tumor; and capillary haemangioma (not
lymphangioma) responds to intralesional corticosteroids.
Lymphangioma usually presents in childhood and can be:
 Anterior: bluish lesion (venous blood); size increases with Valsalva
 Posterior: slowly progressive proptosis which may increase suddenly if there is a spontaneous
bleed; typically followed by ecchymosis.

A 6-year-old girl presents with unilateral blurred vision. On examination, you discover features in
keeping with neuroretinitis with a swollen optic nerve and macular star.

What initial treatment would you offer?

co-amoxiclav

observation

sulfa antibiotics

oral steroids

laser photocoagulation

Your answer was CORRECT


Explanation
Neuroretinitis can be idiopathic or it can be associated with viral infections (mumps, influenza,
varicella) or other infections such as Cat-scratch and leptospirosis. The natural course is for
spontaneous resolution over several months with excellent prognosis; observation is therefore
appropriate initially.

Which one of the following statements regarding Coats' disease is TRUE?


it is usually diagnosed in the second decade
it is usually unilateral
it is autosomal dominant with variable penetrance
females are affected more frequently than males

Your answer was CORRECT


Explanation
Coat's disease is characterised by:
 idiopathic, non-hereditary occurrence
 retinal telangiectasis
 intraretinal and subretinal exudation
 exudative retinal detachment (two-thirds of cases)
 males > females
 bimodal with most presenting in childhood before age 10 yrs
 presenting signs: leucocoria, strabismus, reduced vision,
 90% unilateral
 treatment with Argon laser if progressive exudation or VR if detachment

The best prophylactic treatment for ophthalmia neonatorum is:

viroptic drops

sulfacetamide drops

erythromycin ointment

silver nitrate solution

Your answer was CORRECT


Explanation
Silver nitrate was traditionally used but causes a reactive conjunctivitis. Erythromycin ointment is
more comfortable and is the preferred choice.

A 4-year-old child is referred by her paediatrician with a 1-week history of fever and bilateral
conjunctival injection. Other findings include: a rash, erythema and oedema of the palms and
soles, periungual desquamation, cervical lymphadenopathy, infected pharynx and tongue, and
fissuring of the lips. Ophthalmic evaluation reveals a mild bilateral anterior uveitis.

Which one of the following is the most appropriate treatment for this condition?

aspirin
oral erythromycin

iv penicillin

iv methylprednisolone bolus

oral prednisolone

Your answer was CORRECT


Explanation
The findings suggest Kawasaki disease, a systemic vasculitis of unknown aetiology affecting the
skin, mucous membranes, and heart. The most serious complication, vasculitis, involves the
coronary arteries resulting in formation of coronary artery aneurysms. Aspirin is the treatment of
choice for patients with Kawasaki disease. Systemic corticosteroid therapy use is associated with
an increased rate of coronary artery aneurysm formation and is therefore contraindicated.

What is the MOST frequent indication for penetrating keratoplasty in children?

bacterial keratitis

sclerocornea
interstitial keratitis

Peters' anomaly

Your answer was CORRECT


Explanation

Figure: Peters' anomaly

Peters' anomaly is the most frequent indication for penetrating keratoplasty in children.

All of the following are features of Hutchinson's triad EXCEPT:


interstitial keratitis
saddle nose
deafness
peg-shaped teeth

Your answer was CORRECT


Explanation
Saddle-nose is a recongnised feature of syphilis but does not form part of Hutchinson's triad,
which consists of:
 interstitial keratitis
 peg-shaped teeth
 deafness
Which of the following tests of stereopsis does NOT require wearing special glasses?

TNO

Titmus

Keeler

Lang

Your answer was CORRECT


Explanation
The Lang test uses built-in cylindrical elements to create a 3-dimensional image for measuring
stereopsis (1200 - 600 seconds).

The Frisby uses a glass plate to measure stereopsis and therefore does not require special glasses
to be worn either.

TNO uses red-green glasses for measuring stereopsis (480 - 15 seconds)

Titmus uses Polaroid glasses (3000 - 100 seconds)

Keeler is not a stereopsis test.

A 6-year-old presents with a painless, bluish mass on the nasal quadrant of the upper lid. The
lesion enlarges whenever she coughs.

What is the most likely diagnosis?


orbital varices
capillary haemagioma
cavernous haemangioma
lymphangioma

Your answer was CORRECT


Explanation
Figure: orbital lymphangioma

Lymphangioma usually presents in childhood and can be:


 Anterior: bluish lesion (venous blood); size may increase with Valsalva
 Posterior: slowly progressive proptosis which may increase suddenly if there is a spontaneous
bleed.
Note that primary orbital varices usually have no external signs unless accentuated by a Valsalva
manoeuvre.

Reference: Paediatric Ophthalmology and Strabismus, edited by Kenneth Wright and Peter
Speigel. Springer Science (2013).

Which of the following test results in a newborn would, if positive, BEST support the diagnosis of
congenital toxoplasma infection?
anti-Toxoplasma IgM
liver function tests

MRI brain

anti-Toxoplasma IgG

Your answer was CORRECT


Explanation
IgG against Toxoplasma in the infant's serum is probably maternal and may reflect old maternal
infection. IgM, on the other hand, does not cross the placenta and probably represents a response
to fetal infection. CT scan evidence of intracranial calcification is also very strong.

An 8-year-old boy complains of poor left vision. Fundoscopy reveals a large area of retinal
telangiectasia in the supero-temporal quadrant. There is massive intraretinal exudate in the same
area extending to the macula. The fellow eye is normal.

What is the most likely diagnosis?


retinoblastoma
Coat's disease
Eales disease
familial exudative vitreoretinopathy
Toxocariasis

Your answer was CORRECT


Explanation
Figure: Coat's disease

The features described in the question above are most in keeping with Coat's disease, which is
characterised by:
 idiopathic, non-hereditary occurrence
 retinal telangiectasis
 intraretinal and subretinal exudation
 exudative retinal detachment
 males > females
 bimodal age distribution: childhood or early adulthood with most presenting in childhood
 90% unilateral
 treatment with Argon laser if progressive exudation or VR if detachment
An important differential in the question above is familial exudative vitreoretinopathy (FEVR)
which can also present with extensive exudation leading to retinal detachment. However, FEVR is
usually bilateral and causes temporal dragging of the vascular arcades.

In which of the following causes of neonatal conjunctivitis can the diagnosis be made by Gram
stain alone?

Crede's prophylaxis

Neisseria gonorrhoeae

Chlamydiae subtypes D through K


HSV

Your answer was CORRECT


Explanation
Gonorrhoeae is a Gram negative diplococcus.

A woman, who had retinoblastoma, has a first child also affected with the condition. She would
like to have a second child with the same partner and enquires about the risk of a further child
having the condition.

What should you respond?

50%

100%

40%
2%

7%

Your answer was CORRECT


Explanation

The case presented in the question is one of familial, hereditary retinoblastoma. Familial RB is
inherited in an autosomal dominant fashion, with high penetrance (80%). The risk to a future child
is approximately 40% (not 50% as might be expected for an AD condition because penetrance is
high but not complete).

Which statement is FALSE regarding retinoblastoma:


it appears macroscopically as a smooth white mass

metastatic dissemination is usually via the lymphatics

it is due to mutation in a tumour suppressor gene

it may be successfully treated with radiotherapy and chemotherapy

Your answer was CORRECT


Explanation
Metastatic dissemination of retinoblastoma is via the blood stream. Other options are true.

You might also like